Download as pdf or txt
Download as pdf or txt
You are on page 1of 71

DPPs BOOKLET-1 VIJETA (01-02JPA) | MATHEMATICS

DPP No. # A1 (JEE–ADVANCED)


Total Marks : 37 Max. Time : 30 min.
Single choice Objective ('-1' negative marking) Q.1 to Q.2 (3 marks 3 min.) [06, 06]
Multiple choice objective ('-2' negative & Partial marking) Q.3 to 9 (4 marks 3 min.) [28, 21]
Subjective Questions ('–1' negative marking) Q.10 (3 marks 3 min.) [03, 03]

Question No. 1 2 3 4 5 6 7 8 9 10 Total

Marks Obtained

1. Let A and B two sets containing 2 elements and 4 elements respectively. The number of subsets of
A × B having 3 or more elements is
(A) 256 (B) 220 (C) 219 (D) 211

n
sin(3 r  )
2. If P = (tan (3 n +1) – tan ) and Q =  cos(3r 1 ) , then &
r 0

(A) P = 2Q (B) P = 3Q (C) 2P = Q (D) 3P = Q

3. Which of the follwoing is true :


(A) Remainder when 798 is divided by 5 is 4.
(B) 6n – 5n always leaves the remainder 1 when divided by 25.
(C) Last three digits of the number (27)27 is 803.
(D) (9950 + 10050) < (101)50

4. The coefficient of x98 in the expression of (x – 1) (x – 2) ......... (x – 100) is


(A) 12 + 22 + 32 + ....... + 1002
(B) (1 + 2 + 3 + ....... + 100) 2 – (12 + 22 + 32 + ....... + 1002)
1
(C) [(1 + 2 + 3 + ....... + 100)2 – (12 + 22 + 32 + ....... + 1002)]
2
 
1   100  101 
2
100  101 201
(D)   –
2   2  6 
 

5. The equation (3 + cos x)2 = 4 – 2 sin8x has


(A) exactly one solution in x  (0, 3) (B) exactly three solutions
(C) exactly two solutions x  [0, 5) (D) infinite solutions

6. If quadratic equation x2 + 2(a + 2b)x + (2a + b – 1) = 0 has unequal real roots for all b  R then the
possible values of a can be equal to
(A) 5 (B) – 1 (C) – 10 (D) 3

ab bc c a
7. If = = , then
x y z
abc ab  bc  ca c a b
(A) = (B) = =
xyz ay  bz  cx yz–x xz–y xy–z
c a b a b ab x
(C) = = (D) =
x y z bc bc y

Reg & Corp. Office : CG Tower, A-46 & 52, IPIA, Near City Mall, Jhalawar Road, Kota (Raj.)-324005
Website: www.resonance.ac.in | E-mail : contact@resonance.ac.in
PAGE NO.-1
Toll Free : 1800 258 5555 | CIN: U80302RJ2007PLC024029
DPPs BOOKLET-1 VIJETA (01-02JPA) | MATHEMATICS
8. Consider the function f(x) = mx2 + (2m – 1) x + (m – 2)
(A) If f(x) > 0 x  R then m    ,  
1
 4
(B) The number of integral values of 'm' greater than – 5 so that f(x) < 0  x  R are 4
(C) The number of integral values of m less than 50 so that the roots of the quadratic equation f(x) = 0
are rational, are 6
1
(D) The curve y = f(x) touches the X-axis for m = –
4

9. If a, b, c are natural number satisfying a2 + b2 = c2, then :


(A) at least one of a, b, c is even. (B) at most two of a, b, c can be odd.
(C) abc is divisible by 30. (D) abc is divisible by 8.

10. Find integral values of x for which x2 + 9x + 1 is a perfect square of an integer.

DPP No. # A2 (JEE–ADVANCED)


Total Marks : 35 Max. Time : 30 min.
Single choice Objective ('-1' negative marking) Q.1 to Q.3 (3 marks 3 min.) [09, 09]
Multiple choice objective ('-2' negative & Partial marking) Q.4 to 8 (4 marks 3 min.) [20, 15]
Subjective Questions ('–1' negative marking) Q.9,10 (3 marks 3 min.) [06, 06]

Question No. 1 2 3 4 5 6 7 8 9 10 Total

Marks Obtained

1. Co-efficient of t in the expansion of,


( + p)m  1 + ( + p)m  2 ( + q) + ( + p)m  3 ( + q)2 + ...... ( + q)m  1 , where  q and p  q is

(A)
m

Ct pt  qt  (B)
m

Ct pm  t  qm  t  (C)
m

Ct pt  qt  (D)
m

Ct pm  t  qm  t 
pq pq pq pq

2. Number of roots of equation 3|x|– |2 – |x|| = 1 is


(A) 0 (B) 2 (C) 4 (D) 7

3. Consider the functions f1(x) = 2log2x and f2(x) = log2 x2. If D1, D2 and R1, R2 be the domains and ranges
of the two functions respectively, then
(A) D1= D2 and R1= R2 (B) D1  D2 and R1= R2
(C) D1= D2 and R1  R2 (D) D1  D2 and R1  R2

20
3 1 
4. In the expansion of  4  4 
 6
(A) the number of irrational terms is 19 (B) middle term is irrational
(C) the number of rational terms is 2 (D) 9th term is rational

5. If (x1, y1) & (x2, y2) are the solutions of the equations, log225(x) + log64(y) = 4 and
logx(225) – logy(64) = 1, then
(A) log225x1. log225x2 = 4 (B) log225x1 + log225x2 = 6
(C) |log64y1 – log64y2| = 2 5 (D) log30 (x1x2y1y2) = 12

6. For positive real numbers a (a > 1), let pa and qa be the maximum and minimum values, respectively, of
1
loga(x) for a  x  2a. If pa – qa = , then the value of a is not greater than
2
(A) 3 (B) 4 (C) 5 (D) 6

Reg & Corp. Office : CG Tower, A-46 & 52, IPIA, Near City Mall, Jhalawar Road, Kota (Raj.)-324005
Website: www.resonance.ac.in | E-mail : contact@resonance.ac.in
PAGE NO.-2
Toll Free : 1800 258 5555 | CIN: U80302RJ2007PLC024029
DPPs BOOKLET-1 VIJETA (01-02JPA) | MATHEMATICS
1
7. For the expression (x  R) .
sin x  cos6 x
6

(A) Maximum value is 4 (B) Minimum value is 1


(C) Maximum value is 6 (D) Minimum value is 1/2

8. Number of ways in which 5 colours be selected out of 8 different colours including red, blue and green
(1) if blue and green are always to be included is m
(2) if red is always excluded is n
(3) if red & blue are always included but green excluded is p
(A) m = 20 (B) n = 21 (C) p = 10 (D) None of these

9. Determine the smallest positive value of x (in degrees) for which
tan(x + 100°) = tan(x + 50°) tan x tan (x – 50°).

10. If sin x + sin y  cos  cos x,  x R, then sin y + cos  is equal to

DPP No. # A3 (JEE–ADVANCED)


Total Marks : 37 Max. Time : 30 min.
Single choice Objective ('-1' negative marking) Q. 1 to Q.2 (3 marks 3 min.) [06, 06]
Multiple choice objective ('-2' negative & Partial marking) Q.3 to Q.9 (4 marks 3 min.) [28, 21]
Subjective Questions ('–1' negative marking) Q.10 (3 marks 3 min.) [03, 03]
Question No. 1 2 3 4 5 6 7 8 9 10 Total
Marks Obtained

1. Consider the function f(x) = (x + 1C2x – 8) (2x – 8Cx + 1)


Statement - 1 : Domain of f(x) is singleton.
Statement - 2 : Range of f(x) is singleton.
(A) STATEMENT-1 is True, STATEMENT-2 is True ; STATEMENT-2 is a correct explanation for
STATEMENT-1
(B) STATEMENT-1 is True, STATEMENT-2 is True ; STATEMENT-2 is NOT a correct explanation for
STATEMENT-1
(C) STATEMENT-1 is True, STATEMENT-2 is False
(D) STATEMENT-1 is False, STATEMENT-2 is True

1
2. If the expansion in powers of x of the function is
1– ax  1– bx 
a0 + a1x + a2x2 + a3x3 + ......., then an is :
an – bn an  1 – bn  1 bn  1 – an  1 bn – an
(A) (B) (C) (D)
b–a b–a b–a b–a

3. Numbers which are not in the range of f(x)  x 4  5x 2  4 , are


x(x3  2x 2  x  2)
(A) 1 (B) 2 (C) 3 (D) –1

4. Suppose a, b, c  I such that greatest common divisor of x 2 + ax + b and x2 + bx + c is (x + 1) and


the least common multiple of x2 + ax + b and x2 + bx + c is (x3 – 4x2 + x + 6). Then
(A) a = 1 (B) a = –1 (C) b = –2 (D) c = – 3
.
5. The number of ways in which four different letters can be put in their four addressed envelopes such
that
(A) at least two of them are in the wrong envelopes, are 23
(B) at least two of them are in the wrong envelopes, are 25
(C) exactly two of them are in the wrong envelopes, are 6
(D) exactly two of them are in the wrong envelopes, are 7

Reg & Corp. Office : CG Tower, A-46 & 52, IPIA, Near City Mall, Jhalawar Road, Kota (Raj.)-324005
Website: www.resonance.ac.in | E-mail : contact@resonance.ac.in
PAGE NO.-3
Toll Free : 1800 258 5555 | CIN: U80302RJ2007PLC024029
DPPs BOOKLET-1 VIJETA (01-02JPA) | MATHEMATICS
6. If N = 15246, then :
(A) total number of divisors of N is 36
(B) total number of divisors of N in form of (2n + 1) is 12 (n  Natural)
(C) The number of ways in which N can be resolved as product of two factors is 18
(D) The number of ways in which N can be resolved as product of two coprime factors is 8.

1 1 2
7. The solution set of the inequality –  is (–, ]  (, )  [, ), then
x–2 x x2
(A) +++ = 5 (B)  = –4 (C)  = –2 (D) = 0

8. Solution of
(A) [x]  2 x = 4 is   4,  9 , 4 , 7  where [ . ] represents greatest integer function and { . }
 2 2
represent fractional part function.
(B) [x – 1] + [1 – x] + x – {x} > 0 is x  {1} U [2, ), where [ . ] represents greatest integer function and
{ . } represent fractional part function.
 (C) [x]  2 x = 4 is   4,  9 , 5 , 7  where [ . ] represents greatest integer function and { . }
 2 2
represent fractional part function.
(D) [x – 1] + [1 – x] + x – {x} > 0 is x  {1} U [–2, ), where [ . ] represents greatest integer function
and { . } represent fractional part function.

9. Consider the inequality max {1 – x2, |x – 1|} < 1 then


(A) Solution set is (0, 2) (B) Solution set is (0, 5)
(C) Solution set contain only one integer (D) least integral value satisfying the inequality is 1

10. Four friends A, B, C and D are passing a ball to each other randomly. If A starts passing the ball and 'K'
K
is the total number of ways such that after the 5th pass, the ball reaches back to A, then the value of
2
is ('A' can receive the ball in between as well).

DPP No. # A4 (JEE–MAIN)


Total Marks : 30 Max. Time : 60 min.
Single choice Objective ('-1' negative marking) Q. 1 to Q.19 (3 marks 3 min.) [57, 57]
Subjective Questions ('–1' negative marking) Q.20 (3 marks 3 min.) [03, 03]

Question No. 1 2 3 4 5 6 7 8 9 10 Total


Marks Obtained
Question No. 11 12 13 14 15 16 17 18 19 20 Total
Marks Obtained

1. The sum of the series 20C0 – 20C1 + 20C2 – 20C


3+ ..... + 20C10 is
1 20
(A) –20C10 (B) C10 (C) 0 (D) 20C10
2

2. For some natural N, the number of positive integral 'x' satisfying the equation
1! + 2! + 3! + ...... + x ! = (N)2 is
(A) one (B) two (C) three (D) None of these

3. If letters of the word “PARKAR” are written down in all possible manner as they are in a dictionary, then
the rank of the word ‘PARKAR’ is
(A) 98 (B) 99 (C) 100 (D) 101

4. Sum of all the 4-digit numbers which can be formed using the digits 0, 3, 6, 9 (without repetition of
digits) is
(A) 119988 (B) 115992 (C) 3996 (D) None of these
Reg & Corp. Office : CG Tower, A-46 & 52, IPIA, Near City Mall, Jhalawar Road, Kota (Raj.)-324005
Website: www.resonance.ac.in | E-mail : contact@resonance.ac.in
PAGE NO.-4
Toll Free : 1800 258 5555 | CIN: U80302RJ2007PLC024029
DPPs BOOKLET-1 VIJETA (01-02JPA) | MATHEMATICS
5. 10 10 10 10
If values a, b, c,...... j, p occurs with frequencies C0 , C1, C2 ,..., C10 then mode is
(A) a (B) e (C) f (D) k

6. Sum of coefficients of odd powers of x in the expansion of (1 – x + x2)n is equal to :


3n  1  3n  1 3n  1
(A) (B) (C) (D) None of these
2 2 2

12
 1 
7. In the expansion of  2x   , which of the following term is independent from x ?
 3 x
(A) 9th term (B) 8th term (C) 10th term (D) None of these

8. 40C
0 + 40C1 + …… + 40C20 =
1 40 1 40
(A) 240 (B) 239 + 40C20 (C) 239 – . C20 (D) 239 + . C20
2 2

9. Let n(A) = 5, n(B) = 3 and n(A B) = 2. If number of relations defined from set A to set B is x and
x
number of functional mapping from set A to set B is y, then
y
15 1 1 215
(A) (B) (C) (D)
35 34 54 35

10. If mean of squares of deviations of a set of n observations about –2 and 2 are 18 and 10 respectively
then standard deviation of this set of observations is
(A) 3 (B) 2 (C) 1 (D) None of these

11. The median of a set of 9 distinct observations is 20.5.If each of the largest 4 observations of the set is
increased by 2, then the median of the new set
(A) is decreased by 2 (B) is two times the original median
(C) remains the same as that of the original set (D) is increased by 2

12. Write the negation of the statement "If yuvraj plays then India wins the worldcup".
(A) It is false that India win the worldcup and yuvraj plays
(B) It is false that yuvraj does not play or India win the worldcup
(C) yuvraj plays or India does not win the world cup
(D) yuvraj does not plays and India wins the world cup

13. The equivalent statement of p  q is :


(A) (p  q)  (p  q) (B) (p  q)  (q  p)
(C) (~ p  q)  (p  ~ q) (D) (~ p  q)  (p  ~ q)

14. Let A = {, , } and B = {1, 2} are two sets. P(X) and n(X) represents the power set of X and cardinal
number of set X respectively.
STATEMENT-1 : n(P(A)) + n(P(B)) +n(P(P(A))) + n(P(P(B))) = 284
STATEMENT-2 : If a set X has n elements then number of subsets of set X are 2n.
(A) Statement-1 is True, Statement-2 is True; Statement-2 is a correct explanation for Statement-1.
(B) Statement-1 is True, Statement-2 is True; Statement-2 is NOT a correct explanation for Statement-1
(C) Statement-1 is True, Statement-2 is False
(D) Statement-1 is False, Statement-2 is True

15. Let A = {1, 2, 3, .......n} be a set containing n elements, then for any given k  n, the number of subsets
of A having k as largest element must be
(A) 2k (B) 2k – 1 (C) 2k + 1 (D) 2n – k

Reg & Corp. Office : CG Tower, A-46 & 52, IPIA, Near City Mall, Jhalawar Road, Kota (Raj.)-324005
Website: www.resonance.ac.in | E-mail : contact@resonance.ac.in
PAGE NO.-5
Toll Free : 1800 258 5555 | CIN: U80302RJ2007PLC024029
DPPs BOOKLET-1 VIJETA (01-02JPA) | MATHEMATICS
16. If A is the set of even natural numbers less than 8 and B is the set of prime numbers less than 7, then
the number of relations from A to B is
(A) 29 (B) 92 (C) 32 (D) 29 – 1

17. Let A = {1, 2}, B = {0} then which of the following is correct
(A) Number of possible relations from A to B is 2º =1
(B) Number of void relations from A to B is not possible
(C) Number of possible relations from A to B are 4
(D) Number of possible relations are equal to 2n(A) + n(B)

2 sec 2 x  2 sec x  1
18. Least value of function f(x) = is :
sec 2 x  sec x  5
1 2 5
(A) 2 (B) (C) (D)
5 19 7

19. The solution set of the equation |2x + 3| – |x – 1| = 6, is


(A) x  (–10, 2) (B) x  [–10, 2) (C) x  [–10, 2] (D) x  {–10, 2}

20. Find the number of 4 digit numbers whose product of digits is divisible by 7.

DPP No. # A5 (JEE–ADVANCED)


Total Marks : 38 Max. Time : 30 min.
Single choice Objective ('-1' negative marking) Q. 1 (3 marks 3 min.) [03, 03]
Multiple choice objective ('-2' negative & Partial marking) Q.2 to Q.9 (4 marks 3 min.) [32, 24]
Subjective Questions ('–1' negative marking) Q.10 (3 marks 3 min.) [03, 03]

Question No. 1 2 3 4 5 6 7 8 9 10 Total


Marks Obtained

1. Let f(x) = [9x – 3x + 1] for all x  (–, 1], then the range of f(x) is ; ([ . ] denotes the greatest integer
function)
(A) {0, 1, 2, 3, 4, 5, 6, 7} (B) {0, 1, 2, 3, 4, 5, 6}
(C) {1, 2, 3, 4, 5, 6, 7} (D) {1, 2, 3, 4, 5, 6}

3
2. For x, a > 0 the root(s) of the equation logaxa + logxa2 + loga2 x a = 0 is (are) given by

(A) a–4/3 (B) a–3/4 (C) a–1/2 (D) a–2/3

1
3. The domain of the function f (x) = contains
10
Cx 1  3.10 Cx
(A) All natural numbers (B) Two natural numbers
(C) Two rational numbers (D) Three rational numbers

4. Pair of functions which are identical is/are -

x2  1 2
(A) f(x) = 1  x 2 , g(x) = 1 x 1 x (B) f(x) = sin2x + cos2x, g(x) = +
x 1
2
x 1
2

(C) f(x) = tan2x. sin2x, g(x) = tan2x – sin2x (D) f(x) = sin–1 (sinx), g(x) = cos–1 (cosx)
Reg & Corp. Office : CG Tower, A-46 & 52, IPIA, Near City Mall, Jhalawar Road, Kota (Raj.)-324005
Website: www.resonance.ac.in | E-mail : contact@resonance.ac.in
PAGE NO.-6
Toll Free : 1800 258 5555 | CIN: U80302RJ2007PLC024029
DPPs BOOKLET-1 VIJETA (01-02JPA) | MATHEMATICS
5. A shopkeeper places before you 41 different toys out of which 20 toys are to be purchased. Suppose
m = number of ways in which 20 toys can be purchased without any restriction and n = number of ways
in which a particular toy is to be always included in each selection of 20 toys, then (m – n) can be
expressed as

10 220 1.3.5......19 
(A) 2 (1 . 3 . 5 ....... 39) (B)
20! 10!
19
 4r  2 
(C)   (20 – r) 
r 0
 21   22   23 
(D)    
 40 
 .......  20 
 1  2  3   

6. The no. of ways in which 5 different books to be distributed among 3 persons so that each person gets
at least one book, is equal to the number of ways in which
(A) 5 persons are alloted 3 different residential flats such that each person is alloted at most one flat
and no two persons are alloted the same flat.
(B) No. of parallelograms formed by one set of 6 parallel lines and other set of 5 parallel lines that goes
in other direction.
(C) 5 different toys are to be distributed among 3 children, so that each child gets at least one toy.
(D) None of these

   
2
  
1 cos   
7. Values of ‘x’ satisfying the inequality |2x2 – 4x – 7| < 1     , –   < where [ . ]
 2   
 cos  sin  
2 2
  2 2 
 
denotes the greatest integer function, is

(A) (–1, 3) 
(B) 1  5, 1  5  (C) (1  5, 1) (D) (3 , 1  5 )

8. Given the function f(x) such that


   x 
2 f(x) + xf 
1
  2f  2 sin    x  1    = 4 cos2 + x cos
x    4   2 x
 
(A) f(2) + f(1/2) = 1
(B) f(1) =  1, but the values of f(2), f(1/2) cannot be determined
(C) f(2) + f(1) = f(1/2)
(D) f(2) + f(1) = 0

9. If (1+x)15 = C0 + C1. x + C2. x2 +.... + C15. x15 and the value of C2 + 2C3 + 3C4 +.... + 14C15 =.214 + .
(Where  &  are the natural numbers), then
(A)  = 13 (B)  +  = 16 + 215 (C)  +  = 14 (D)  = 215 – 1

 
10. Let R and   
k 2
sin 2k  = a. Find the value  cot  2   cot  2  sin 2  in terms of 'a'.
k 0
3 k k 4 k

Reg & Corp. Office : CG Tower, A-46 & 52, IPIA, Near City Mall, Jhalawar Road, Kota (Raj.)-324005
Website: www.resonance.ac.in | E-mail : contact@resonance.ac.in
PAGE NO.-7
Toll Free : 1800 258 5555 | CIN: U80302RJ2007PLC024029
DPPs BOOKLET-1 VIJETA (01-02JPA) | MATHEMATICS

DPP No. # A6 (JEE–ADVANCED)


Total Marks : 37 Max. Time : 30 min.
Single choice Objective ('-1' negative marking) Q. 1 to Q.2 (3 marks 3 min.) [06, 06]
Multiple choice objective ('-2' negative & Partial marking) Q.3 to Q.9 (4 marks 3 min.) [28, 21]
Subjective Questions ('–1' negative marking) Q.10 (3 marks 3 min.) [03, 03]

Question No. 1 2 3 4 5 6 7 8 9 10 Total


Marks Obtained

1. Let tan (2 |sin |) = cot (2 |cos |), where   R and f(x) = (|sin | + |cos |)x, x  1.
Then range of f(x) does not include
(A) 1 (B) 2 (C) 3 (D) 4

2. Let  be distinct real number such that


a2 + b + c = (sin ) 2 + (cos )
a2 + b + c = (sin ) 2 + (cos )
a2 + b + c = (sin ) 2 + (cos ) (where a, b, c,  R)
a2  b2
The maximum value of the expression is equal to
a2  3ab  5b2
(A) 1 (B) 2 (C) 3 (D) 4

3. All the natural numbers, sum of whose digits is 8 are arranged in ascending order then the 170th
number in the list does not contain the digit
(A) 2  (B) 3 (C) 4 (D) 5

4. Consider a triangle ABC and let a, b and c denote the lengths of the side opposite to vertices A, B and
C respectively. If a = 1, b = 3 and C = 60º, then
7 4 7
(A) c = 7 (B) c  7 (C) R  (D) r 
3 2 3

5. If logx (2 + x)  logx (6 – x) then x can be


(A) (1, 2] (B) (0, 1)  (1, 2] (C) (0, 1)  [2, 6) (D) (3/2, 2]

12
6. If B  and A  1  2  5  10 , then value of logAB is
3 5  8
(A) a positive integer (B) a prime integer
(C) a non-negative integer (D) non integer

x2  x 2x  1 x3
7. If x is real and (x) = 3x  1 2  x 2
x 3  3 = a0x7 + a1x6 + a2x5 + ... + a6x + a7 then
x3 x2  4 2x
6
(A) a7 = 21 (B) a
k 0
k = 111 (C)  (–1) = –32 (D) (1) = 121

8. If 25C 25C +2. 25C 25C +3. 25C . 25C + . . . . + 24 . 25C . 25C =k. 49C + 50C , then the value of
0 2 1 3 2 4 23 25  

2k –  –  is greater than (where , < 25)


(A) 1 (B) 2 (C) 3 (D) 4
Reg & Corp. Office : CG Tower, A-46 & 52, IPIA, Near City Mall, Jhalawar Road, Kota (Raj.)-324005
Website: www.resonance.ac.in | E-mail : contact@resonance.ac.in
PAGE NO.-8
Toll Free : 1800 258 5555 | CIN: U80302RJ2007PLC024029
DPPs BOOKLET-1 VIJETA (01-02JPA) | MATHEMATICS
9. Different words are formed by arranging the letters of the word ''SUCCESS'', then
(A) The number of words in which C are together but S's are seperated, is 24
(B) The number of words in which no two C's and no two S's are together is 96
(C) The number of words in which the consonants appear in alphabetic order is 42
(D) None of these

tan(x – )  tan x  tan(x  )


10. Number of values of  in [0, 2] for which the expression y = is
tan(x – ) tan x tan(x  )
independent of x.
 ax2  bx  c a b c
 You may use the fact that, the ratio is independent of x if   
 px2  qx  r p q r 

DPP No. # A7 (JEE–ADVANCED)


Total Marks : 37 Max. Time : 30 min.
Single choice Objective ('-1' negative marking) Q. 1 (3 marks 3 min.) [03, 03]
Multiple choice objective ('-2' negative & Partial marking) Q.2 to Q.8 (4 marks 3 min.) [28, 21]
Subjective Questions ('–1' negative marking) Q.9 to Q.10 (3 marks 3 min.) [06, 06]

Question No. 1 2 3 4 5 6 7 8 9 10 Total


Marks Obtained

1. If the graph of the function y = f(x) is as shown :

–2
x
0 1 2

–1
then the graph of y = 1/2( |f(x)| – f(x)) is
y y
1 1
(A) (B)
x x
–2 0 1 2 –2 0 1 2

y y
1
1
(C) x (D)
–2 –1 0 1 2 –2 0 1 2
x

–1
2. If |sin2x + 17 – x2| = |16 – x2| + 2sin2x + cos2x then subsets of solution are
(A) {0} (B) [– 4, 4] (C) [– 8, 8] (D)  17, 17 
 

3. The coefficient of x4 in the expansion of (1  x + 2x2)12 is:


(A) 12C3 (B) 13C3 (C) 12C8 + 412C10 + 6 12C9 (D) 12C3+ 3 13C3 + 14C4

4. If in the expansion of (1 + x)m (1– x)n, the coefficients of x and x2 are 3 and – 6 respectively, then
(A) n = 6 (B) n = 9 (C) m = 12 (D) m = 24
21  4a  a2
5. The solution set satisfying the inequality,  1 contains :
a1
(A) [ 7,  1) (B) [2, 3] (C) [2, ) (D) [ 7, 0)

Reg & Corp. Office : CG Tower, A-46 & 52, IPIA, Near City Mall, Jhalawar Road, Kota (Raj.)-324005
Website: www.resonance.ac.in | E-mail : contact@resonance.ac.in
PAGE NO.-9
Toll Free : 1800 258 5555 | CIN: U80302RJ2007PLC024029
DPPs BOOKLET-1 VIJETA (01-02JPA) | MATHEMATICS
6. The equation |x + 2| – |x + 1| + |x – 1| = K, x  R has two solutions if K is equal to
(A) 4 (B) 3/2 (C) 5/2 (D) 3

7. P(x) is a polynomial with integral cofficient such that for four distinct integers
a, b, c, d; P(a) = P(b) = P(c) = P(d) = 3.If P(e) = 5 (e is an integer), then e can't be
(A) 1 (B) 3 (C) 4 (D) 5

8. Let ordered pair () satisfying the system of equations


2 log (x2 + y2) – log5 = log {2(x2 + y2) + 75} and log  x  + log(5y) = 1 + log 2 then
3
(A) number of such ordered pair is 2 (B) 2
+ 2 = 25
(C)  &  are prime numbers (D)  &  are coprime numbers

9. Find the number of quadratic polynomials ax2 + 2bx + c which satisfy the following conditions :
(i) a, b, c are distinct
(ii) a, b, c  {1, 2, 3, ........... 2001, 2002}
(iii) x + 1 divides ax2 + 2bx + c

10. Consider the word W = "COMMISSIONER". Find


(i) Number of 5 letters word containing two vowels and three consonants.
(ii) Number of ways in which all the letters of the word W can be arranged if alike letters are
together but seperated from the other alike letters.
(iii) Number of ways in which letters of the word W can be arranged without changing order of alike
letters.

DPP No. # A8 (JEE–MAIN)


Total Marks : 60 Max. Time : 60 min.
Single choice Objective ('-1' negative marking) Q. 1 to Q.20 (3 marks 3 min.) [60, 60]

Question No. 1 2 3 4 5 6 7 8 9 10
Marks Obtained
Question No. 11 12 13 14 15 16 17 18 19 20 Total
Marks Obtained

1. If x  2  22 / 3  21/ 3 , then the value of (x 3  6x 2  6x) is


(A) 3 (B) 2 (C) 1 (D) None of these

2. Number of 4 digit positive integers if the product of their digits is divisible by 3, is


(A) 2700 (B) 6628 (C) 7704 (D) 5464

3. There are 9 straight lines of which 5 are concurrent at a point and other 4 are concurrent at another
point and no two of these 9 lines are parallel then number points of intersection is
(A) 20 (B) 22 (C) 36 (D) 38

4. The exponent of 12 in 100! is


(A) 97 (B) 58 (C) 48 (D) None of these

5. The total number of non-negative integer 'n' satisfying the equations n2 = p + q and n3 = p2 + q2, where
p and q are integers, is
(A) 1 (B) 2 (C) 3 (D) Infinite

Reg & Corp. Office : CG Tower, A-46 & 52, IPIA, Near City Mall, Jhalawar Road, Kota (Raj.)-324005
Website: www.resonance.ac.in | E-mail : contact@resonance.ac.in
PAGE NO.-10
Toll Free : 1800 258 5555 | CIN: U80302RJ2007PLC024029
DPPs BOOKLET-1 VIJETA (01-02JPA) | MATHEMATICS
x y
6. If a = and b = . The value of (x + y) is
x y
2 2
x  y2
2

(a  b)2 ab
(D) a  b
ab 2 2
(A) (B) (C)
a2  b2 a b
2 2 a b
2 2

7. If log45 = x and log56 = y then


1 1
(A) log46 = xy (B) log64 = xy (C) log32 = (D) log23 =
2xy  1 2xy  1

8. If 4x4 + 9y4 = C6 then the maximum value of xy is


C2 C3 C2 C3.
(A) (B) (C) (D)
2 3 2 3 3 2 3 2

9. If (1 + x + x² +... + xp)n = a0 + a1x + a2x²+...+anp xnp, then the value of a1 + 2a2 + 3a3 +.... + np anp is
n n
 p  1 n  p  1 np
(A) np   (B) (p + 1)n (C) n   (D) (p + 1)n
 2  2  2  2

10. Which of the following is logically equivalent to ~ (p  q)


(A) (~ p)  q (B) (~ p)  (~q) (C) p  (~q) (D) p  q

11.In an experiment with 15 observations on x, the following results were available


x2 = 2830, x = 170.
One observation that was 20 was found to be wrong and was replaced by the correct value 30. Then
the corrected variance is
(A) 80.33 (B) 78.00 (C) 188.66 (D) 177.33

12. The mean deviation from mean of the data 6, 7, 10, 12, 13, 4, 8, 20 is
(A) 3.75 (B) 3.5 (C) 4.5 (D) 3.55

13. When a polynomial P(x) is divided by x, (x – 2) and (x – 3), remainders are 1, 3 and 2 respectively. If
the same polynomial is divided by x(x – 2)(x – 3), the remainder is g(x), then the value of g(1) is
(A) 1 (B) 5/3 (C) 2/3 (D) 8/3

14. Product of roots of the equation (logax)3 – 12 (logax)2 + 44 loga x – 48 = 0 is 64, then a is
(A) 2 (B) 3 (C) 2 (D) 3

15. Consider the following relations


R1 = { (x , y) : x, y are integers and x = ay or y = ax for some integer a }
R2 = {(x, y) : x, y are integers and ax + by = 1 for some integers a, b}
Then
(A) R2 is an equivalence relation but R1 is not (B) R1, R2 are not equivalence relations.
(C) R1, R2 are equivalence relations. (D) R1 is an equivalence relation but R2 is not

16. Which of the following is not an equivalence relation


(A) aRb  a = b for some rational  where a,b  Z+
(B) aRb  a = b for some rational  where a,b  Z–
(C) aRb  a = b for some rational  where a,b  Z
(D) aRb  a2 = b2 for a,b  Z

17. If domain of f(x) is [0, 1], then domain of f({x}3 + 1), where {.} represents fractional part function, is
(A) (– , 0) (B) { x : x = k, k  I }
(C) Set of all rational numbers (D) { x : k  x < k + 1/2, k  I }
Reg & Corp. Office : CG Tower, A-46 & 52, IPIA, Near City Mall, Jhalawar Road, Kota (Raj.)-324005
Website: www.resonance.ac.in | E-mail : contact@resonance.ac.in
PAGE NO.-11
Toll Free : 1800 258 5555 | CIN: U80302RJ2007PLC024029
DPPs BOOKLET-1 VIJETA (01-02JPA) | MATHEMATICS
18. Let f : R  R be a function defined by f(x) = [x]2 + [x + 1] – 3 {where [.] denontes greatest integer
function}, then f(x) is
(A) many-one into (B) many-one onto (C) one-one into (D) one-one onto

19. Which of the following function is neither even nor odd.


x 1 (a x  1)5
(A) f(x) =      sin x (B) f(x) = ,a>0
 2  ax
x x
(C) f(x) = + +1
e 1 2
x

g(x)  g(  x)
(D) f(x) = , where g(x) is a real valued function x  R
5

20. About the dolls in a shop a customer said "It is not true that some dolls have neither black hair nor blue
eyes". The customer means that :
(A) all dolls have both black hair and blue eyes
(B) all dolls have black hair but not blue eyes
(C) some dolls have either black hair or blue eyes
(D) all dolls have either black hair or blue eyes

DPP No. # A9 (JEE–ADVANCED)


Total Marks : 34 Max. Time : 29 min.
Single choice Objective ('-1' negative marking) Q. 1 (3 marks 3 min.) [03, 03]
Multiple choice objective ('-2' negative & Partial marking) Q.2 to Q.6 (4 marks 3 min.) [20, 15]
Subjective Questions ('–1' negative marking) Q.7 (3 marks 3 min.) [03, 03]
Match the Following (each row '(2, -1)' negative marking) Q.8 (8 marks, 8 min.) [08, 08]

Question No. 1 2 3 4 5 6 7 8 9 10 Total


Marks Obtained

1. Number of zeros after decimal before first significant digit in the number (0. 3 )10000 is
(Given log103 = 0.4771)
(A) 4770 (B) 4771 (C) 4772 (D) 4773

2. If the sum of digits of the number N = 200011 –2011 is S, then


(A) S is a prime number
(B) Sum of digits of S is 10
(C) (S+1) is divisible by exactly 3 prime numbers
(D) S is a composite number

3. If n is the smallest positive integer such that polynomial x4 – nx + 63 can be written as a product of a
linear and a cubic polynomial with integer coefficicents, then
(A) Number of divisors of n is 10.
(B) Sum of digits of n is 12.
(C) Number of natural solutions of xyz = n is 45.
(D) Number of odd divisors of n is 2.

4. If ,  are the roots of the equation x2 + x +  = 0 such that   and |x – | –  < , then
(A) inequality is satisfied by exactly two integral values of x
(B) inequality is satisfied by all values of x  (– 4, – 2)
(C) Roots of the equation are opposite in sign
(D) x2 + x +  < 0  x  [–1, 0]

Reg & Corp. Office : CG Tower, A-46 & 52, IPIA, Near City Mall, Jhalawar Road, Kota (Raj.)-324005
Website: www.resonance.ac.in | E-mail : contact@resonance.ac.in
PAGE NO.-12
Toll Free : 1800 258 5555 | CIN: U80302RJ2007PLC024029
DPPs BOOKLET-1 VIJETA (01-02JPA) | MATHEMATICS

If f  2x  , 2x   = xy, then f(m, n) + f(n, m) is


y y
5.
 8 8
(A) depends over m and n both (B) periodic and odd function
(C) constant number (D) even function

6. If the remainder R(x) = ax + b is obtained by dividing the polynomial x 100 by the polynomial x2 – 3x + 2
then
(A) a = 2100 – 1 (B) b = 2(299 – 1) (C) b = – 2(299 – 1) (D) a = 2100

7. If domain of f(x) be (–1, 2) then


(B) domain of f(n x) will be  , e 2 
1
(A) domain of f(sin x) will be (–)
e  
(C) domain of f([x]) is [0, 2) (D) domain of f({x}) is R
(where [.] and {.} are integral part and fractional part functions respectively)

8. If the solution of the equation |(x4–9) —(x2 + 3)| = |x4 – 9| – |x2 + 3| is (–, p]  [q, ) then
(A) p + q = 0 (B) |p| + |q| = 4 (C) |q – p| = 4 (D) |pq| = 4

9. Given that x + siny = 2008, cosy + x = 2007, where 0  y /2. find the value of x + y.

1 x 1
10. Let f(x) = x + and g(x) = .
x x2
Match the composite function given in Column–I with respective domains given in Column–II.
Column I Column II
(A) fog(x) (p) R – {–2, –5/3}
(B) gof(x) (q) R – {–1,0}
(C) fof(x) (r) R – {0}
(D) gog(x) (s) R – {–2, –1}
(t) R – {– 1}

DPP No. # A10 (JEE–ADVANCED)


Total Marks : 39 Max. Time : 35 min.
Single choice Objective ('-1' negative marking) Q. 1 to Q.3 (3 marks 3 min.) [09, 09]
Multiple choice objective ('-2' negative & Partial marking) Q.4 to Q.7 (4 marks 3 min.) [16, 12]
Subjective Questions ('–1' negative marking) Q.8,9 (3 marks 3 min.) [06, 06]
Match the Following (each row '(2, -1)' negative marking) Q.10 (8 marks, 8 min.) [08, 08]

Question No. 1 2 3 4 5 6 7 8 9 10 Total


Marks Obtained

1. Number of permutations of 1, 2, 3, 4, 5, 6, 7, 8 and 9 taken all at a time, such that the digit
1 appearing somewhere to the left of 2
3 appearing to the left of 4 and
5 somewhere to the left of 6, is
(e.g. 815723946 would be one such permutation)
(A) 9 · 7! (B) 8! (C) 5! · 4! (D) 8! · 4!
2n n
2. If (1 + 2x +3x2)n = 
r 0
ar x r , then a
r 1
2r =

(A) 6n (B) (3n + 1).2n–1 (C) (3n + 1).2n – 1 (D) (3n + 1).2n–1 – 1

3. If the equation sin (cot–1 (cos (tan–1x))) =  has a solution, then  lies in the interval
 1   1 1 
(A) (0, 1] (B)  , 1 (C)  – ,  (D) (–, 1]
 2   2 2

Reg & Corp. Office : CG Tower, A-46 & 52, IPIA, Near City Mall, Jhalawar Road, Kota (Raj.)-324005
Website: www.resonance.ac.in | E-mail : contact@resonance.ac.in
PAGE NO.-13
Toll Free : 1800 258 5555 | CIN: U80302RJ2007PLC024029
DPPs BOOKLET-1 VIJETA (01-02JPA) | MATHEMATICS
4. Which of the following equations have no real solutions ?
(A) x2 – 2x + 5 + x = 0 (B) log1.5(cot–1x–sgn(ex)) = 2
x
(C) x4 – 2x2sin2 +1=0 (D) all of these
2

5. For the equation log3 x x  log3x x  0 , which of the following do not hold good?
(A) no real solution (B) one prime solution
(C) one integral solution (D) no irrational solution

6. Which of the following is a rational number?



(A) sin  tan1 3  tan1  (B) cos 
1 3
 sin1
 3 2 4 

 1 63   1 1 5

(C) log2  sin  sin1   (D) tan  cos 
 4
 8 
 2 3 

7. For the function f(x) = 4  x2 + x2  1


(A) Domain is [–2, 2] (B) Domain is [–2, –1]  [1, 2]
(C) Range is [ 3, 5 ] (D) Range is [ 3 , 6 ]

8. Find the number of triplets (a, b, c) such that a  b  c and a! + b! + c! = 3d, where a, b, c, d N.

12
9. Find the number of solutions of the equation  sinrx  0 lying in the interval (0, ].
r 1

10. Column-I Column-II


(A) Let f (x) be a function on (– , ) and f (x + 2) = f (x – 2). If f (x) = 0 has (P) 4
only three real roots in [0, 4] and one of them is 4, then the number of real roots
of f (x) = 0 in [– 8, 10], can be

(B) Let r1, r2, r3........, rn be n positive integers, not necessarily distinct, such that (Q) 5
(x + r1)(x + r2) ....(x + rn) = xn + 56xn – 1 + ...... + 2009
The possible value of n is

(C) If x and y are positive integers and 2xy = 2009 – 3y, then the number of (R) 8
ordered pairs of (x,y), is

(x  4)2 y 2
(D) If x, y  R, satisfying the equation   1 then the difference (S) 9
4 9
x2 y2
between the largest and smallest value of the expression  , is
4 9

Reg & Corp. Office : CG Tower, A-46 & 52, IPIA, Near City Mall, Jhalawar Road, Kota (Raj.)-324005
Website: www.resonance.ac.in | E-mail : contact@resonance.ac.in
PAGE NO.-14
Toll Free : 1800 258 5555 | CIN: U80302RJ2007PLC024029
DPPs BOOKLET-1 VIJETA (01-02JPA) | MATHEMATICS

DPP No. # A11 (JEE–ADVANCED)


Total Marks : 33 Max. Time : 30 min.
Comprehension ('–1' negative marking) Q. 1 to Q.3 (3 marks 3 min.) [09, 09]
Single choice Objective ('-1' negative marking) Q.4 to Q.7 (3 marks 3 min.) [12, 12]
Multiple choice objective ('-2' negative & Partial marking) Q.8 to Q.10 (4 marks 3 min.) [12, 09]

Question No. 1 2 3 4 5 6 7 8 9 10 Total


Marks Obtained

Comprehension # (1 to 3)
x 1
f(x)  ; g ( x)  , h(x) = 2x + 1
x 1 1 x
 
fn (x)  f f  f.........f(x))  ....... , n N, similarly gn(x) and hn(x) are defined
n times

1 1
1. Number of values of 'n' for which the equation   g6 (x) , has no solution, is 
fn (x) x
  (A) 0 (B) 1 (C) 2 (D) n – 1 
2. Solution of the equation hn(x) = g3n(x) will lie in the domain of
(A) g  f(x)  (B) h  f(x)  (C) g h(x) (D) f  h(x) 

3. Triangle formed by the lines y = h2(x) , y = g3(x) and y = f4(x) (4x + 1)+ x + 2 is
(A) right angle triangle (B) obtuse angle triangle
(C) acute angle triangle (D) lines are concurrent hence no triangle is formed

4. Let A = {1, 2………., 10} and B = {1, 2,……., 5}


f : A  B is a non-decreasing into function, then number of such functions is
(A) 1001 (B) 1876 (C) 205 (D) 875

5. If ,  and  are the three angles with  = 2 tan–1  


2 1 ;  = 3 sin–1  1 
 2
–1  1 
 + sin    and
 2

  = cos–1  1  , then
3
(A)  >  >  (B)  >  >  (C)  >  >  (D)  =  > 

6. If f(x) = x11 + x9 – x7 + x3 + 1 and f(sin–1 (sin 8)) = ,  is constant, then f(tan –1 (tan 8)) is equal to
(A)  (B) 2 –  (C)  – 2 (D)  + 2

 1  50   31  
7. The value of sec  sin   sin   cos1 cos    is equal to
  9   9 
10  
(A) sec (B) sec (C) 1 (D) – 1
9 9
2x 2
8. If a, b respectively be the numbers of solutions and sum of solutions of – |x| = x , then
x 1 | x  1|
(A) a = 3 (B) b = 1 (C) b = 2 (D) a = 2

9. Which of the following is/are correct?


17
(A) tan cos –1  tan–1  = (B) cos  tan –1
4 2 1 1 1
 tan –1  =
 5 3 6  3 2 2
 –1  1   14
(D) cos  2cos –1
1 1 2 6
(C) sin  2tan    + cos(tan–1 2 2 ) =  sin–1  = –
  3  15  5 5 5
Reg & Corp. Office : CG Tower, A-46 & 52, IPIA, Near City Mall, Jhalawar Road, Kota (Raj.)-324005
Website: www.resonance.ac.in | E-mail : contact@resonance.ac.in
PAGE NO.-15
Toll Free : 1800 258 5555 | CIN: U80302RJ2007PLC024029
DPPs BOOKLET-1 VIJETA (01-02JPA) | MATHEMATICS
1 1 x x 1
10. Let f1(x) = x, f2(x) = 1 – x, f3(x) = , f4(x) = , f5(x) = , f6(x) = . If f6(fm(x)) = f4(x) and
x 1 x x 1 x
fn(f4(x)) = f3(x), then :
(A) m = 5 (B) n = 5 (C) m = 6 (D) n = 6

DPP No. # A12 (JEE–ADVANCED)


Revision DPP on "Binomial Theorem "
Total Marks : 30 Max. Time : 30 min.
Single choice Objective ('-1' negative marking) Q. 1 to Q.6 (3 marks 3 min.) [18, 18]
Subjective Questions ('–1' negative marking) Q.7 to Q.10 (3 marks 3 min.) [12, 12]

Question No. 1 2 3 4 5 6 7 8 9 10 Total


Marks Obtained

m
 1 
STATEMENT - 1 : The term independent of x in the expansion of  x   2  is
(2m) !
1.
 x  (m !)2
STATEMENT - 2 :The coefficient of xb in the expansion of (1 + x)n is nCb.
(A) STATEMENT-1 is true, STATEMENT-2 is true and STATEMENT-2 is correct explanation for
STATEMENT-1
(B) STATEMENT-1 is true, STATEMENT-2 is true and STATEMENT-2 is not correct explanation
for STATEMENT-1
(C) STATEMENT-1 is true, STATEMENT-2 is false
(D) STATEMENT-1 is false, STATEMENT-2 is true
(E) Both STATEMENTS are false

2n 2n
2. If 
r 0
ar (x  1)r   b (x  2)
r 0
r
r
and br = (–1)r – n for all r  n, then an =

(A) 2n + 1Cn – 1 (B) 3nCn (C) 2n + 1Cn (D) 0

N
3. If r 1
(r2 + 2r + 1). r! = (2014)! –2, then N can be

(A) 2012 (B) 2013 (C) 2011 (D) 2014

m  10   20  p
4. The sum    
 i  m 
 , (where   = 0, if p < q) is maximum when ' m ' is
i  q
i0
(A) 5 (B) 10 (C) 15 (D) 20

n
5. The sum 
r0
(r + 1) Cr2 is equal to :

n  2 2n  1! n  2 2n  1! n  2 2n  1! n  2 2n  1!


n! n  1 ! n! n 1! n! n  1! n! n  1!
(A) (B) (C) (D)

n 1 3r 7r 
6. If 
r 0
( 1)r. nCr 
 2
r

22 r

23 r
 ........to m terms = k  1  1  , then k =
  2m n 
1 1 1 1
(A) (B) (C) (D)
2 1
n
2 2n
1 2 2n
1 2 1
n

Reg & Corp. Office : CG Tower, A-46 & 52, IPIA, Near City Mall, Jhalawar Road, Kota (Raj.)-324005
Website: www.resonance.ac.in | E-mail : contact@resonance.ac.in
PAGE NO.-16
Toll Free : 1800 258 5555 | CIN: U80302RJ2007PLC024029
DPPs BOOKLET-1 VIJETA (01-02JPA) | MATHEMATICS
(4n  1)!
7. Prove that, (2nC1)²+ 2. (2nC2)² + 3. (2nC3)² +... + 2n. (2nC2n)² =
{(2n  1)!} 2

n n  n   n  1 n n  2 k  


n n  k  n
8.Prove that 2k     – 2k 1    + 2
k 2
    –...... + (– 1)    =   .
0 k    
1 k  1   
2 k  2     0  k 
k

n n
9. For any positive integer m, n (with n  m), let   = Cm. Prove that
m
n  n  1  n  1 m  n 1
  +    +... +   =   . Hence or otherwise, prove that
 
m  m  m   
m  m  1
n  n  1 n  2 m n2
 + 2 + 3  +... + (n – m + 1)   =  .
m  m   m  m m  2

10. Find the coefficient of x50 in the expression:


(1 + x)1000 + 2x (1 + x)999 + 3x² (1 + x)998 +..... + 1001 x1000

DPP No. # A13 (JEE–ADVANCED)


Total Marks : 30 Max. Time : 27 min.
Single choice Objective ('-1' negative marking) Q. 1 to Q.4 (3 marks 3 min.) [12, 12]
Multiple choice objective ('-2' negative & Partial marking) Q.5 to Q.9 (4 marks 3 min.) [12, 09]
Subjective Questions ('–1' negative marking) Q.10 (3 marks 3 min.) [06, 06]

Question No. 1 2 3 4 5 6 7 8 9 10 Total


Marks Obtained

1. A variable line L = 0 is drawn through O(0, 0) to meet the lines L 1: x + 2y – 3 = 0 and
L2 : x + 2y + 4 = 0 at points M and N respectively. A point P is taken on L = 0 such that
1 1 1
2
 2
 , then locus of point P is
OP OM ON2
144 144 144 144
(A) x2 + 4y2 = (B) (x + 2y)2 = (D) 4x2 + y2 = (D) (x – 2y)2 =
25 25 25 25

2. The co-efficient of xn  2 in the polynomial (x  1) (x  2) (x  3) ....... (x  n) is :


n (n2  2) (3n  1) n (n2  1) (3n  2)
(A) (B)
24 24
n (n  1) (3n  4)
2
(C) (D) none of these
24


3. Let f : R  0 ,  –1 2
 defined by f(x) = tan (x + x + a), then the set of values of 'a' for which f is onto
 2 
is
1
(B)  ,  
1
(A) [0, ) (C) (D) (0 , )

4  4

 1  x2 
4. Total number of solution of the equation cos–1   = sin–1x is/are
 1 x
2

(A) one (B) two (C) three (D) four

Reg & Corp. Office : CG Tower, A-46 & 52, IPIA, Near City Mall, Jhalawar Road, Kota (Raj.)-324005
Website: www.resonance.ac.in | E-mail : contact@resonance.ac.in
PAGE NO.-17
Toll Free : 1800 258 5555 | CIN: U80302RJ2007PLC024029
DPPs BOOKLET-1 VIJETA (01-02JPA) | MATHEMATICS

If 2 tan–1x + sin–1 
2x 
5.  is independent of 'x', then value of 'x' can be
 1  x2 

1
(A) 2 (B) (C) – 1 (D) any real number
2

6. If f(x) = x3 + 2x2 + 3x + 4 and g(x) is the inverse of f(x) then

1 1 4 4
(A) g' ( 4)  (B) g' (4)  (C) g" ( 4)  (D) g" ( 4) 
4 3 27 27

7. Which of the following quantities is/are positive?


(A) cos(tan–1(tan4)) (B) sin(cos–1(cot4)) (C) tan(cos–1(cos5)) (D) cot(sin–1(sin4))

8. If f(x) = sin  [x  5]  x  x  {x}    for x   0,




4 
is invertible, where {.} and [.] represent

fractional part and greatest integer functions respectively, then f –1(x) is



(A) sin–1x (B) – cos–1x (C) sin–1{x} (D) cos–1{x}
2

9 Two finite sets have m and n elements. The number of subsets of the first set is 112 more than that of
the second set. The values of m and n are
(A) m = 7 (B) n = 4 (C) m = 3 (D) n = 8

10 In a right angled triangle, acute angles A and B satisfy


tan A + tan B + tan2A + tan2B + tan3A + tan3B = 70
find the angle A and B in radians.

DPP No. # A14 (JEE–MAIN)


Total Marks : 61 Max. Time : 60 min.
Single choice Objective ('-1' negative marking) Q. 1 to Q.19 (3 marks 3 min.) [57, 57]
Multiple choice objective ('-2' negative & Partial marking) Q.20 (4 marks 3 min.) [04, 03]

Question No. 1 2 3 4 5 6 7 8 9 10 Total


Marks Obtained
Question No. 11 12 13 14 15 16 17 18 19 20 Total
Marks Obtained

1. The number of integral solutions of the inequation x + y + z  100, (x  2, y  3, z  4), is


(A) 102C2 (B) 94C3 (C) 93C2 (D) None of these

Reg & Corp. Office : CG Tower, A-46 & 52, IPIA, Near City Mall, Jhalawar Road, Kota (Raj.)-324005
Website: www.resonance.ac.in | E-mail : contact@resonance.ac.in
PAGE NO.-18
Toll Free : 1800 258 5555 | CIN: U80302RJ2007PLC024029
DPPs BOOKLET-1 VIJETA (01-02JPA) | MATHEMATICS
2. Study carefully the graph of a certain function.
y

(–1, 4) (1, 4)

3 y = f(x)

–3 –1 0 1 3

The graph corresponds to


(A) y = ||x|2 – |x| – 6| (B) |y| = ||x|2 – 2|x| – 3|
(C) y = ||x|2 – 2|x| – 3| (D) |y| = ||x|2 – |x| – 6|

 1 1 1  1 2  1 3  1 1999 
3. The sum of   +   +  +  + ...... +  
2 2 2000   2 2000   2 2000  2 2000 
where [ . ] denotes the greatest integer function, is equal to :
(A) 1000 (B) 999 (C) 1001 (D) 1002

n1 99
4. Ifr 0
log2  r  2  =
 r 1
log (r + 1), then ‘n’ is equal to
r 10
r

(A) 4 (B) 3 (C) 5 (D) 6

5. Let N be the number of quadratic equations with coefficients from {0, 1, 2, 3...., 9} such that 0 is a
solution of each equation. Then the value of N is
(A) 29 (B) infinite (C) 90 (D) 900

6. 116 people participated in a knockout tennis tournament. The players are paired up in the first round,
the winners of the first round are paired up in the second round, and so on till the final is played
between two players. If after any round, there is odd number of players, one player is given a bye, i.e.
he skips that round and plays the next round with the winners. The total number of matches played in
the tournament is
(A) 115 (B) 53 (C) 232 (D) 116

7. Number of 4 digit numbers of the form N = abcd which satisfy following three conditions
(i) 4000  N < 6000 (ii) N is a multiple of 5 (iii) 3  b < c  6 is equal to
(A) 12 (B) 18 (C) 24 (D) 48

8. If a new chess game is invented which has 9 × 9 grid, tiled by 81 squares of size 1 × 1 , in chess board
pattern, instead of 8 × 8 and rule is that in a single step the king can only move along horizontal or
vertical to the adjacent tile. If king is at the center tile of this grid then find total number of shortest paths
he can take to reach at any corner of chess board ?
(A) 280 (B) 140 (C) 560 (D) none

 1 x 2  3x  2  is
Domain of the function f(x) = log  sin
9. 
 
 3 5 3 5 
(A) (–, –2)  (–1, ) (B)   , 
 2 2 
 
 3  5   3 5
(C)  ,  2     1,  (D) none of these
 2   2 
 

Reg & Corp. Office : CG Tower, A-46 & 52, IPIA, Near City Mall, Jhalawar Road, Kota (Raj.)-324005
Website: www.resonance.ac.in | E-mail : contact@resonance.ac.in
PAGE NO.-19
Toll Free : 1800 258 5555 | CIN: U80302RJ2007PLC024029
DPPs BOOKLET-1 VIJETA (01-02JPA) | MATHEMATICS
10. A survey shows that 60% of the people watch a News channel, whereas 76% watch another channel. If
x % of the people watch both are channels, then
(A) x = 35 (B) x = 63 (C) 36  x 60 (D) x = 39

11. Number of solutions of the equation, cot–1 (x2 + 1) = sin–1 |x|


(A) one (B) two (C) three (D) None of these

 2  1  2  5
12. If  = tan–1   – tan–1   and  = tan-1(3) – sin–1   , then cot ( – )
 2 – 1  2   5 
(A) is equal to 1 (B) is equal to 0 (C) is equal to 2 – 1 (D) is non existent

Number of solutions of the equation sin  cos1 x  = 1 is


1
13.
3 
(A) only one (B) No solution (C) only three (D) at least two

14. Let R1 = {(m, n)  Z × Z : (m + 4n) is divisible by 5 in Z}.


and R2 = {(m, n)  Z × Z : (m + 9n) is divisible by 5 in Z} where Z be the set of integers.
Which one of the following is correct ?
(A) R1 is a proper subset of R2 (B) R2 is a proper subset of R1
(C) R1 = R2 (D) R1 is not a symmetric relation on Z

15. Which one of the following is not a contradiction?


(A) [~p  (p  ~q)]  q (B) (~p  q)  p
(C) [(p  q)  p]  ~p (D) (~q  ~p)  (p  q)

16. If the mean of the set of numbers x1, x2, ......., xn is x then the mean of the numbers xi + 2i, 1  i  n is
(A) x  2n (B) x  n  1 (C) x  2 (D) x  n

17. Sum of squares of deviation of 10 observations measured from 5 is 17 and sum of squares of
observations is 170 then mean of observation is
(A) 40.3 (B) 4.5 (C) 4 (D) 4.03

18. If cos1 x + cos1 y + cos1 z = 0 , then which of the following is correct ?


(A) x + y + z = 0 (B) x2 + y2 + z2 = x y + y z + z x
1 1 1
(C) sin x + sin y + sin z = 0 (D) tan1 x + tan1 y + tan1 z = 0

19. Which of the following is NOT true for any two statements p and q ?
(A) ~ [p v (~ q)] = (~ p)  q (B) ~ (p v q) = (~ p) v (~q)
(C) q  ~q is a contradiction (D) ~ (p  (~ p)) is a tautology

20. Which of the following are CORRECT ?


(A) R : {(x, y)| x ,y are real numbers and x = wy for some rational number w} is an equivalence relation
m p
(B) S = {  ,  | m, n, p and q are integers such that n, q  0 and qm = pn} is an equivalence relation
 n q
(C) Let W denote the words in the english dictionary. Define the relation T by
: T = {(x, y)  W × W | the words x and y have at least one letter in common} then T is an equivalence
relation
(D) Let A = N × N be the Certesian product of N and N. A relation P is defined as
P = { (m, n), (p, q)  A × A: m + q = n + p}, then P is an equivalence relation.

Reg & Corp. Office : CG Tower, A-46 & 52, IPIA, Near City Mall, Jhalawar Road, Kota (Raj.)-324005
Website: www.resonance.ac.in | E-mail : contact@resonance.ac.in
PAGE NO.-20
Toll Free : 1800 258 5555 | CIN: U80302RJ2007PLC024029
DPPs BOOKLET-1 VIJETA (01-02JPA) | MATHEMATICS

DPP No. # A15 (JEE–ADVANCED)


Total Marks : 27 Max. Time : 35 min.
Comprehension ('–1' negative marking) Q. 1 to Q.3 (3 marks 3 min.) [09, 09]
Single choice Objective ('-1' negative marking) Q. 4 to Q.6 (3 marks 3 min.) [09, 09]
Multiple choice objective ('-2' negative & Partial marking) Q.7 to Q.8 (4 marks 3 min.) [08, 06]
Subjective Questions ('–1' negative marking) Q.9 (3 marks 3 min.) [03, 03]
Match the Following (each row '(2, -1)' negative marking) Q.10 (8 marks, 8 min.) [08, 08]

Question No. 1 2 3 4 5 6 7 8 9 10 Total


Marks Obtained

Comprehension # (For Q.1 to Q.3)

In the figure below, it is given that C = 90°, AD = DB, ED is


perpendicular to AB, AB = 20 units and AC = 12 units.
C
E



 
A D B

1. Area of triangle AEC is


21
(A) 24 sq. units (B) 21 sq. units (C) 42 sq. units (D) sq. units
2
2. The value of tan ( + )is
117 17 3 5
(A) – (B) (C) (D)
44 4 4 4
3. The value of cos ( + ), is
4 3 117 44
(A) (B) (C) (D) –
5 5 125 125

50
–1  
 tan
2n –1
4. The sum  4  equals tan K, where K equal
n 1  n – n  1
2

(A) 5050 (B) 2550 (C) 4950 (D) 2525

5. If f(x) + f(1–x) = 2 and g(x) = f(x)–1 x  R, then g(x) is symmetric about
1
(D) the point  , 0 
1
(A) the origin (B) the line x = (C) the point (1, 0)
2 2  

7
2 r 
6. The value of  tan  16  is
r 1
(A) 29 (B) 33 (C) 34 (D) 35

Reg & Corp. Office : CG Tower, A-46 & 52, IPIA, Near City Mall, Jhalawar Road, Kota (Raj.)-324005
Website: www.resonance.ac.in | E-mail : contact@resonance.ac.in
PAGE NO.-21
Toll Free : 1800 258 5555 | CIN: U80302RJ2007PLC024029
DPPs BOOKLET-1 VIJETA (01-02JPA) | MATHEMATICS
7. The vertices of a triangle are A(x1, x1 tan ), B(x2, x2 tan ) and C(x3, x3 tan ). If the circumcentre of
a
triangle ABC coincides with the origin and H(a, b) be the orthocentre, then =
b
x1  x 2  x 3 x cos   x 2 cos   x 3 cos 
(A) (B) 1
x1 tan   x 2 tan   x 3 tan  x1 sin   x 2 sin   x3 sin 
tan   tan   tan  cos   cos   cos 
(C) (D)
tan  .tan  .tan  sin   sin   sin 

8. Let f(x) be invertible function. If equation f(f –1(x)) = f–1(x) has two distinct real roots  and , then
(A) f(x) = x also have same two real roots
(B) f–1(x) = x also have same two real roots
(C) f(x) = f–1(x) also have same two real roots
(D) Area of triangle formed by (0, 0), (, f()) and (, f()) is 1 square unit
9. Determine the number of ways of walking from (0, 0) to (5, 5) using only up and right unit steps such
that the path does not pass through any of the following points (1, 1), (1, 4), (4, 1), (4, 4)

10. Match the entries of column–I with entries of column–II if domain of function in column-I is equal to
range of function in column–II
Column– Column–
(A) f(x) = cos–1 
2x 
2 
(P) f(x) = x3 + x2 – x + 1
 1 x 
x2  x  1
(B) f(x) = ln (tan–1(x2 + 1)) (Q) f(x) =
x2  x  1

(C) f(x) = 10x  3x 2  3 (R) f(x) = ln x


3x  5  5  2(cos x  3 sin x)
(D) f(x) = cos–1   (S) f(x) =
 4  3

DPP No. # A16 (JEE–ADVANCED)


Total Marks : 41 Max. Time : 35 min.
Single choice Objective ('-1' negative marking) Q. 1 to Q.3 (3 marks 3 min.) [09, 09]
Multiple choice objective ('-2' negative & Partial marking) Q.4 to Q.9 (4 marks 3 min.) [24, 18]
Match the Following (each row '(2, -1)' negative marking) Q.10 (8 marks, 8 min.) [08, 08]
Question No. 1 2 3 4 5 6 7 8 9 10 Total
Marks Obtained

1. There is an equilateral triangle with side 4 and a circle with the centre on one of the vertex of that
triangle. The arc of that circle divides the triangle into two parts of equal area. How long is the radius of
the circle?
12 3 24 3 30 3 6 3
(A) (B) (C) (D)
   

x|x|
2. If f(x) = – , then f–1(x) equals
1  x2
|x| |x| x |x|
(A) (B) (sgn (–x)) (C) – (D) (sgn(x))
1 | x | 1 | x | 1 x 1 |x|
Reg & Corp. Office : CG Tower, A-46 & 52, IPIA, Near City Mall, Jhalawar Road, Kota (Raj.)-324005
Website: www.resonance.ac.in | E-mail : contact@resonance.ac.in
PAGE NO.-22
Toll Free : 1800 258 5555 | CIN: U80302RJ2007PLC024029
DPPs BOOKLET-1 VIJETA (01-02JPA) | MATHEMATICS
3. tan–1n, tan–1(n + 1) and tan–1(n + 2), n  N, are the angles of a triangle if n =
(A) 1 (B) 2 (C) 3 (D) None of these

The sum of cot–1 


7
+ cot–1 
19 
+ cot–1 
39 
+ cot–1 
67 
4.     ....... is equal to
4  4   4   4 
 1
(A) tan–1 4 (B) tan–1 3 (C) tan–1 2 (D)  tan1
2 2

5. Which of the following pair(s) of functions are identical ?


1  x2 2x
(A) f(x) = cos (2 tan–1x), g(x) = (B) f(x) = , g(x) = sin (2 cot–1x)
1 x 2
1  x2
(D) f(x) = e n (sgn cot x) , g(x) = e n [1{x}] ,
1
(C) f(x) = tan x + cot x, g(x) = 2 cosec 2x
where sgn(.), [.], {.} denotes signum, greatest integer and fractional part functions respectively)

 2 
6. If function f(x) = cos–1x + cos–1  x  3 – 3x  , then
2 2 
 
 10    5
(A) Range of f(x) is  ,  (B) Range of f(x) is  , 
3 3  3 3  

(C) f(x) is one-one for x   –1 ,  (D) f(x) is one-one for x   , 1


1 1
 2 2  

7. Let R(x1, y1) and S(x2, y2) be the end points of latus rectum of parabola y2 = 4x. The equation of ellipse
1
with latus rectum RS and eccentricity are (a > b)
2
(3x  1)2 3y 2 (3x  7)2 3y 2
(A) + =1 (B) + =1
64 16 64 16
(3x  1)2 3y 2 (3x  7)2 3y 2
(C) + =1 (D) + =1
8 64 8 8

8. The function f(x) = (tan–1x)3 – (cot–1x)2 + tan–1x + 2 is


(A) decreasing  x  R. (B) Increasing  x  R.
(C) Bounded (D) Many one function.

9. If f(x) = 2 tan 3x + 5 1  cos6x and g(x) is a function having the same period as that of f(x), then which
of the following cannot be g(x)?

(B) 2 1  cos 3x + cosec3x


2
(A) 2 sin3x + 3 cos3x
(C) 3 cosec3x + 2 tan3x (D) (sec23x + cosec23x) tan23x

10. Match the following :


Column - I Column – II

(A) f : R  ,   and f(x) = cot–1 (2x–x2 –2), then f(x) is (p) one-one
4 
(B) f: R  R and f(x) = eax sinbx where a,b, R+, then f(x) is (q) into

(C) f : R+  [2, ) and f(x) = 2 + 3x2 , then f(x) is (r) many-one

(D) f: X  X and f(f(x)) = x  x X, then f(x) is (s) onto

(t) invertible

Reg & Corp. Office : CG Tower, A-46 & 52, IPIA, Near City Mall, Jhalawar Road, Kota (Raj.)-324005
Website: www.resonance.ac.in | E-mail : contact@resonance.ac.in
PAGE NO.-23
Toll Free : 1800 258 5555 | CIN: U80302RJ2007PLC024029
DPPs BOOKLET-1 VIJETA (01-02JPA) | MATHEMATICS

DPP No. # A17 (JEE–ADVANCED)


Total Marks : 41 Max. Time : 35 min.
Single choice Objective ('-1' negative marking) Q. 1 to Q.3 (3 marks 3 min.) [09, 09]
Multiple choice objective ('-2' negative & Partial marking) Q.4 to Q.9 (4 marks 3 min.) [24, 18]
Match the Following (each row '(2, -1)' negative marking) Q.10 (8 marks, 8 min.) [08, 08]

Question No. 1 2 3 4 5 6 7 8 9 10 Total


Marks Obtained

  2  0.25  sincos x 2x 4  + 1 = 0,




2
tan x  
1. The set of values of x satisfying the equation, 2 4 is :
(A) an empty set (B) a singleton
(C) a set containing two values (D) an infinite set

2. If sin (sin x + cos x) = cos(cos x – sin x), then the largest possible value of sin x, is

1 16  2 
(A) (B) 1 (C) (D)
2 4 4

3. Let [x] denote the greatest integer part of a real number x, if


40  2 
n
M    then M equals
n1  2 
(A) 5700 (B) 5720 (C) 5740 (D) 11060

4. If sin2( – ) cos = cos2( – ) sin = m sin cos, then –


1 1
(A) | m |  (B) | m |  (C) |m|  1 (D) |m|  1
2 2
| sin x |  | cos x |
5. The period of function is
| sin x – cos x |  | sin x  cos x |
 2
(A)  (B) (C) 2 (D)
2 3

6. The inequality [2 – x] + 2 [x – 1]  0 is satisfied by (where [.] denotes greatest integer function) :


(A) x  {0} (B) x  W (C) x  N (D) x  [1, )

7. If {.} represents fractional part function and {x} + {–x} = x2 + x – 6, then


(A) integral roots are –2 & 3 (B) number of non-integral roots is 2
(C) number of solutions is 4 (D) equation has exactly two integral roots

21
 r 
8. If f
r 0
 11  2x  = constant  x  R and f(x) is periodic, then period of f(x) is
 

1
(A) 1 (B) (C) 2 (D) 4
11

Reg & Corp. Office : CG Tower, A-46 & 52, IPIA, Near City Mall, Jhalawar Road, Kota (Raj.)-324005
Website: www.resonance.ac.in | E-mail : contact@resonance.ac.in
PAGE NO.-24
Toll Free : 1800 258 5555 | CIN: U80302RJ2007PLC024029
DPPs BOOKLET-1 VIJETA (01-02JPA) | MATHEMATICS
9. Which of the following is/are true ?

(A) number of digits in 812 535 is 35

(B) Number of digits in 812 535 is 36


20
 8 
(C) Number of zeroes after decimal before a significant figure starts in   is 10
 27 
20
 8 
(D) Number of zeroes after decimal before a significant figure starts in   is 11
 27 

10. Column-I Column-II


(A) Suppose sin  – cos  = 1 (P) 0
then the value of sin3 – cos3 is (  R)

(B) Minimum value of the function (Q) 1


f (x) = (1 + sin x)(1 + cos x)  x  R, is

(C) Given that the sum of the solutions of the equation (R) 2/3
sin x · tan x – sin x + tan x – 1 = 0 over [0, 2] = k,
where k  Q then the value of k equals (S) 3/2

sin4 t  cos4 t  1
(D) The expression when simplified reduces to
sin6 t  cos6 t  1

DPP No. # A18 (JEE–ADVANCED)


Revision DPP on "Permutation and Combination"
Total Marks : 32 Max. Time : 30 min.
Comprehension ('–1' negative marking) Q. 1 to Q.3 (3 marks 3 min.) [09, 09]
Single choice Objective ('-1' negative marking) Q.4 to Q.8 (3 marks 3 min.) [15, 15]
Multiple choice objective ('-2' negative & Partial marking) Q.9 to Q.10 (4 marks 3 min.) [08, 06]

Question No. 1 2 3 4 5 6 7 8 9 10 Total


Marks Obtained

Comprehension # (1 to 3)

Consider the word W = SOLICITATION consisting of 6 vowels namely three I's, one A's and two O's
and 6 consonants namely two T's and C, L, N, S one each. Words are formed using only the letters
from the word W.

1. If the number of ways in which all the letters of the word W can be arranged so that vowels appear
alphabetically is K(7!), then the value of K is
(A) 99 (B) 33 (C) 66 (D) 72

2. Number of 4 lettered word that can be made if each word contains exactly 2 consonants and
exactly 2 vowels, is
(A) 1008 (B) 504 (C) 1056 (D) 168

3. If the number of words that can be made by using all the letters of the word W, if O's as well as I's are
separated is N(8!), then the value of N is
(A) 158 (B) 228 (C) 260 (D) None

Reg & Corp. Office : CG Tower, A-46 & 52, IPIA, Near City Mall, Jhalawar Road, Kota (Raj.)-324005
Website: www.resonance.ac.in | E-mail : contact@resonance.ac.in
PAGE NO.-25
Toll Free : 1800 258 5555 | CIN: U80302RJ2007PLC024029
DPPs BOOKLET-1 VIJETA (01-02JPA) | MATHEMATICS
4.Number of triangles with each side having integral length and the longest side is of 11 units, is equal to
(A) 6 (B) 36 (C) 20 (D) 32

5. A staircase has 10 steps. A person can go up the steps one at a time, two at a time or any combination
of 1st and 2nd. Then number of ways in which the person can go up the stairs, is
(A) 66 (B) 89 (C) 68 (D) 93

6. The total number of divisors of the number N = 2 5 . 34 . 510 . 76 that are of the form 4k + 2, K  N is
equal to
(A) 385 (B) 384 (C) 96 (D) 77

7. Triplet (x, y, z) is chosen from the set (1, 2, 3,....,n), such that x  y < z . The number of such triplets
is
(A) n3 (B) nC3 (C) nC2 (D) nC2 + nC3

8. If all the permutations of the letters in the word OBJECT are arranged in alphabetical order as in a
dictionary. The 717th word is
(A) TJOECB (B) TOJECB (C) TOJCBE (D) none of these

9. The number of triangles that can be formed using the vertices of a 20 sided regular polygon such that
the triangle and the polygon does not have any side common is
20
C1 . 16 C2
(A) 20C1 . 16C2 (B) (C) 2400 (D) 800
3

10. If the number of ways in which 7 letters can be put in 7 envelopes such that exactly four letters are in
wrong envelopes is N, then
(A) Number of divisors of N is 12
(B) Sum of proper divisors of N is 624
(C) Number of divisors of N which are divisible by 3 but not by 9 is 4
(D) Sum of proper divisors of N is 309

DPP No. # A19 (JEE–ADVANCED)


Total Marks : 35 Max. Time : 30 min.
Single choice Objective ('-1' negative marking) Q. 1 to Q.4 (3 marks 3 min.) [12, 12]
Multiple choice objective ('-2' negative & Partial marking) Q.5 to Q.9 (4 marks 3 min.) [20, 15]
Subjective Questions ('–1' negative marking) Q.10 (3 marks 3 min.) [03, 03]

Question No. 1 2 3 4 5 6 7 8 9 10 Total


Marks Obtained

1. Let f(x) = xx ; x (1, ) and g(x) be inverse function of f(x) then g (x) must be
1 1 1 1
(A) (B) (C) (D)
x (1  n x) x (1  n (g(x))) g(x).(1  n(g(x)) g( x )(1  nx)

2. A bag contains 50 tickets numbered 1, 2, 3....,50 of which five are drawn at random and arranged in
ascending order of magnitude (x1 < x2 < x3 < x4 < x5). The probability that x3 = 30 is
20
C2 2
C2 20
C2 29 C2
(A) 50 (B) 50 (C) 50 (D) None of these
C5 C5 C5

3. The probability that a positive two digit number selected at random has its tens digit at least three more
than its unit digit is
(A) 14/45 (B) 7/45 (C) 36/45 (D) 1/6

Reg & Corp. Office : CG Tower, A-46 & 52, IPIA, Near City Mall, Jhalawar Road, Kota (Raj.)-324005
Website: www.resonance.ac.in | E-mail : contact@resonance.ac.in
PAGE NO.-26
Toll Free : 1800 258 5555 | CIN: U80302RJ2007PLC024029
DPPs BOOKLET-1 VIJETA (01-02JPA) | MATHEMATICS
4. n different books (n  3) are put at random in a shelf. Among these books there is a particular book 'A'
and a particular book B. The probability that there are exactly 'r' books between A and B is
2 2(n  r  1) 2(n  r  2) (n  r)
(A) (B) (C) (D)
n(n  1) n(n  1) n(n  1) n(n  1)

k n

 = tan–1   , then the value of 'n' is


2k
5. If tan1 6
k 1 2  k2  k4 7
(A) a prime number (B) an odd number (C) an even number (D) divisible by 3

6. Number of 3-digit odd numbers can be formed using the digits 1, 2, 3, 4, 5, 6 if


(A) The repetition of digits is not allowed is 60
(B) The repetition of digits is allowed 208
(C) The repetition of digits is not allowed 40
(D) The repetition of digits is allowed 108

7. There are 720 permutations of the digits 1, 2, 3, 4, 5, 6 suppose these permutations are arranged from
smallest to largest numerical values beginning from 123456 and ending with 654321.
(A) Number falls on the 124th position is 213564
(B) The position of the number 321546 is 267
(C) Number falls on the 124th position is 223564
(D) The position of the number 321546 is 261

8. Given that N = 2n (2n+1 – 1) and 2n+1 – 1 is a prime number, which of the following is true, where n is a
natural number
(A) sum of divisors of N is 2N
(B) sum of reciprocals of divisors of N is 1
(C) sum of the reciprocals of the divisors of N is 2
(D) sum of divisors of N is 4N

k  45
9. If  cosec (2k  1) = m , where m and n are integers more than 1, then
k 1
2 n

(A) m + n = 91
(B) |m – n| = 87
(C) m + n = 82
(D) Number of ways in which n + 5m + 1 can be resolved as product of two factors is 5.

10. If all roots of the equation x4 – 12x3 + bx2 + cx + 81 = 0 are positive and g(x) is inverse of
f(x) = x+2 + ( + 2)x + , where  is a root of 2bx + c = 0, then find the value of 24g'(5).

DPP No. # A20 (JEE–MAIN)


Total Marks : 64 Max. Time : 60 min.
Single choice Objective ('-1' negative marking) Q. 1 to Q.16 (3 marks 3 min.) [48, 48]
Multiple choice objective ('-2' negative & Partial marking) Q.17 to Q.20 (4 marks 3 min.) [16, 12]

Question No. 1 2 3 4 5 6 7 8 9 10 Total


Marks Obtained
Question No. 11 12 13 14 15 16 17 18 19 20 Total
Marks Obtained

1. Number of words each consisting of two vowels and two consonants which can be made out of the
letters of the word ‘DEVASTATION’ is
(A) 126 (B) 198 (C) 1512 (D) 1638

Reg & Corp. Office : CG Tower, A-46 & 52, IPIA, Near City Mall, Jhalawar Road, Kota (Raj.)-324005
Website: www.resonance.ac.in | E-mail : contact@resonance.ac.in
PAGE NO.-27
Toll Free : 1800 258 5555 | CIN: U80302RJ2007PLC024029
DPPs BOOKLET-1 VIJETA (01-02JPA) | MATHEMATICS
2. Number of ways in which 3 tickets can be selected from a set of 500 tickets numbered
1, 2, 3, ......, 500 so that the number on them are in arithmetic progression is
(A) 500 (B) 500C3 (C) 62250 (D) None of these

3. If ‘m’ denotes the number of 5 digit numbers when each successive digits are in their descending order
of magnitude and ‘n’ is the corresponding figure when the digits are in their ascending order of
magnitude, then (m – n) has the value
(A) 2. 10C5 (B) 10C4 (C) 9C3 (D) 9C5
4. If x1 and x2 are the means of two distributions such that x1  x2 and x is the mean of the combined
distribution then
x1  x2
(A) x  x1 (B) x  x2 (C) x  (D) x1  x  x2
2

5. Which of the following is logically equivalent to ~(p q)


(A) ~ p ~ q (B) ~ p q (C) (p ~q)  (q  ~p) (D) ~ (p q)  (q ~ p)

6. Which of the following statement is not equivalent statement conveying same meaning as "If
mathematics is tough then either physics is easy or chemistry is huge".
(A) If physics is not easy and chemistry is not huge then mathematics is not tough.
(B) If it is not true that "physics is easy or chemistry is huge" then mathematics is not tough.
(C) Mathematics is tough, physics is tough and chemistry is not huge.
(D) "Mathematics is tough" is sufficient condition for physics being easy or chemistry being huge.

7. The number of integral value(s) of 'p' for which the equation


99 cos 2 – 20 sin 2 = 20p + 35, will have a solution is
(A) 8 (B) 9 (C) 10 (D) 11

8. Let  be a real number such that 0    . If f(x) = cos x + cos(x + ) + cos(x + 2) takes some
constant number c for any x  R, then the value of [c + ] is equal to
(Note : [y] denotes greatest integer less than or equal to y.)
(A) 0 (B) 1 (C) – 1 (D) 2
4x 2 1
9. If f(x) = 5 . Which of the following are true / False ?
. The domain of f(x) is |x|  1/2  . The range of f(x) is y  1
. The graph of f(x) is symmetric about the y-axis.
IV. The graph of f(x) never decreases
(A)TTTT (B) TFTF (C) TTTF (D) TFTT

 3   11 
10. The value of sin–1   + cos–1   + cot–1 3 is
 73   146 
5 17 7
(A) (B) (C) (D) None of these
12 12 12

11. Total number of ordered pairs (x, y) satisfying |y| = cosx and y = sin –1 (sinx) where x  [–2, 3] is equal
to
(A) 5 (B) 4 (C) 6 (D) 2
Number of solutions of the equation tan – 1 
1 
= tan –1   + tan – 1  2 
1 1
12.  
 a  1 x  a  x  1
(A) one (B) Two (C) Three (D) Zero

cos  sin1  cos 1  =


1 1
13.
 2 3
3 8  3 8 3 8
(A) (B) (C) (D) 0
6 6 6
Reg & Corp. Office : CG Tower, A-46 & 52, IPIA, Near City Mall, Jhalawar Road, Kota (Raj.)-324005
Website: www.resonance.ac.in | E-mail : contact@resonance.ac.in
PAGE NO.-28
Toll Free : 1800 258 5555 | CIN: U80302RJ2007PLC024029
DPPs BOOKLET-1 VIJETA (01-02JPA) | MATHEMATICS

14. A fair die is thrown 3 times. The chance that sum of three numbers appearing on the die is less than 11,
is equal to
1 2 1 5
(A) (B) (C) (D)
2 3 6 8
15. Let a red die, a blue die, a green die and a white die are rolled once, the dice being fair. The outcomes
on the red, blue, green and white die denote the numbers a, b, c and d respectively. Let E denotes the
event that absolute value of (a – 1)(b – 2)(c – 3)(d – 6) = 1, then P(E) is
1 1 2 1
(A) (B) (C) (D)
324 648 324 162

16. When 12 balls are put in three boxes, the probability that the first box will contain 3 balls is
12
29 C 3 29 12
C3 212
(A) (B) (C) (D) None of these
312 312 312

17. Let a distribution be made by combining three distributions, each having mean zero, standard
deviations 3 , 4 and 5 respectively, and frequencies 200 , 250 and 300 respectively. Then the variance
of the combined distribution is equal to
266 50 62
(A) (B) 17 (C) (D)
15 3 15

18. Let S be any set and P(S) be its power set. We define a relation R on P(S) by
ARB  A  B.  A, B  P(S) then relation R is :
(A) Reflexive (B) Symmetric (C) Transitive (D) Equivalence

19. In a certain town 25% families own a phone and 15% own a car, 65% families own neither a phone nor
a car. 2000 families own both a car and a phone, then
(A) 10% families own both a car and a phone
(B) 35% families own either a car or a phone
(C) 40,000 families live in the town
(D) 2000 families own both a car and a phone

20. If A and B are two sets such that A has 12 elements and B has 17 elements and A  B has 21
elements then number of elements in A  B is :
(A) 8 (B) Total number of subsets of {p, q, r}
(C) 7 (D) Total number of proper subsets of {p, q, r}

DPP No. # A21 (JEE–ADVANCED)


Total Marks : 36 Max. Time : 30 min.
Comprehension ('–1' negative marking) Q. 1 to Q.2 (3 marks 3 min.) [06, 06]
Single choice Objective ('-1' negative marking) Q. 3 to Q.4 (3 marks 3 min.) [06, 06]
Multiple choice objective ('-2' negative & Partial marking) Q.5 to Q.10 (4 marks 3 min.) [24, 18]

Question No. 1 2 3 4 5 6 7 8 9 10 Total


Marks Obtained

Comprehension # (Q.1 & 2)

    tan  tan  ; where


2n 2n

 
2n
For x   0, cos cos1 x3r 1 and Tn  1 3r

 4 
, Let Sn   sin sin1 x3r 2 , Cn 
r 1
x
r 1 r 1
n N and n  3.

1. The correct order of Sn , Cn and Tn is given by


(A) Sn > Tn > Cn (B) Sn < Cn < Tn (C) Sn < Tn < Cn (D) Sn > Cn > Tn
Reg & Corp. Office : CG Tower, A-46 & 52, IPIA, Near City Mall, Jhalawar Road, Kota (Raj.)-324005
Website: www.resonance.ac.in | E-mail : contact@resonance.ac.in
PAGE NO.-29
Toll Free : 1800 258 5555 | CIN: U80302RJ2007PLC024029
DPPs BOOKLET-1 VIJETA (01-02JPA) | MATHEMATICS

2. The value of 'x' for which Sn = Cn + Tn, is


   
(A) sin (B) 2sin (C) 2sin (D) sin
5 5 10 10

3. 18 points are indicated on the perimeter of a triangle ABC (see figure).


If three points are choosen, then probability that it will form a triangle :-

B C
331 1 355 711
(A) (B) (C) (D)
816 2 408 816

4. Two non-negative integers are chosen at random, then the probability that the sum of their squares is
divisible by 5 is
7 8 9 5
(A) (B) (C) (D)
25 25 25 25

5. 4 people are selected randomly out of six married couple. Then the probability that

(A) exactly one married couple is formed is 240


12
C4
15
(B) exactly two married couple are formed is
12
C4
240
(C) they do not form a married couple is
12
C4
15
(D) they do not form a married couple is
12
C4

6. There are two groups of subjects one of which consists of 5 science subjects and 3 engineering
subjects and the other consists of 3 science and 5 engineering subjects. An unbiased die is cast. If
number 3 or number 5 turns up, a subject is selected at random from the first group, otherwise the
subject is selected at random from the second group. The probability that an engineering subject is
selected ultimately is P, then
13 1 1 3 3
(A) P = (B) 0 < P < (C) <P< (D) < P <1
24 2 2 4 4

7. If two events A and B are such that P(Ac) = 0.3. P(B) = 0.4 and P(A Bc) = 0.5, then
(A) P(A  BC) = 0.8 (B) P[B  (A  BC)] = 0.2
(C) P(B/A  B ) = 0.25
c (D) P(B/A  Bc) = 0.3

1 1   14   
8. The value of cos  cos  cos     is
 2   5   
2 7   3
(A) cos (B) cos    (C) sin (D) – cos
5  5  10 5

Reg & Corp. Office : CG Tower, A-46 & 52, IPIA, Near City Mall, Jhalawar Road, Kota (Raj.)-324005
Website: www.resonance.ac.in | E-mail : contact@resonance.ac.in
PAGE NO.-30
Toll Free : 1800 258 5555 | CIN: U80302RJ2007PLC024029
DPPs BOOKLET-1 VIJETA (01-02JPA) | MATHEMATICS
9. 3 3
An extremum value of the function f(x) = (arc sin x) + (arc cos x) is:
7 3 3 3 3
(A) (B) (C) (D)
8 8 32 16

10. In a triangle ABC, if a = 4, b = 8 and C = 60º, then which of the following relations is(are) correct?
[Note : All symbols used have usual meaning in triangle ABC.]
(A) The area of triangle ABC is 8 3 (B) The value of  sin2 A = 2
2 3 4
(C) Inradius of triangle ABC is (D) The length of internal angle bisector of angle C is
3 3 3

DPP No. # A22 (JEE–ADVANCED)


Total Marks : 36 Max. Time : 30 min.
Single choice Objective ('-1' negative marking) Q.1 to Q.2 (3 marks 3 min.) [06, 06]
Multiple choice objective ('-2' negative & Partial marking) Q.3 to Q.8 (4 marks 3 min.) [24, 18]
Subjective Questions ('–1' negative marking) Q.9,10 (3 marks 3 min.) [06, 06]
Question No. 1 2 3 4 5 6 7 8 9 10 Total
Marks Obtained

1. There are 4 urns. The first urn contains 1 white and 1 black ball, the second urn contains 2 white and
3 black balls, the third urn contains 3 white and 5 black balls & the fourth urn contains 4 white &
i2  1
7 black balls. The selection of each urn is not equally likely. The probability of selecting i th urn is
34
(i = 1, 2, 3, 4). If we randomly select one of the urns & draw a ball, then the probability of ball being
white is
569 27 8
(A) (B) (C) (D) None of these
1496 56 73

1
2. The sum of the maximum and minimum values of the function f (x) = is
1  (2cos x  4 sin x)2
22 21 22 21
(A) (B) (C) (D)
21 20 20 11

3. The letters of the word PROBABILITY are written down at random in a row. Let E1 denotes the event
that two 's are together and E2 denotes the event that two B's are together, then
3 2 18 1
(A) P(E1) = P(E2) = (B) P(E1  E2) = (C) P(E1  E2) = (D) P(E1/E2) =
11 55 55 5

4. A player tosses a coin. He gets one point for head and two point for tail. He plays till he gets sum of
points equal to n. If pn be the probability that his score becomes n, then
1 1 1 11 1
(A) p3 = (B) pn = pn–1 + pn–2 (C) p4 = (D) pn = (pn–1 + pn–2)
2 2 4 16 2

5. A bag initially contains one red & two blue balls. An experiment consisting of selecting a ball at random,
noting its colour & replacing it together with an additional ball of the same colour. If three such trials are
made, then:
(A) probability that atleast one blue ball is drawn is 0.9
(B) probability that exactly one blue ball is drawn is 0.2
(C) probability that all the drawn balls are red given that all the drawn balls are of same
colour is 0.2
(D) probability that atleast one red ball is drawn is 0.6.

Reg & Corp. Office : CG Tower, A-46 & 52, IPIA, Near City Mall, Jhalawar Road, Kota (Raj.)-324005
Website: www.resonance.ac.in | E-mail : contact@resonance.ac.in
PAGE NO.-31
Toll Free : 1800 258 5555 | CIN: U80302RJ2007PLC024029
DPPs BOOKLET-1 VIJETA (01-02JPA) | MATHEMATICS
6. A Six-faced die is biased such that when it is thrown, the chance of showing an even number is twice
that of an odd number. If it is thrown thrice, then the probability that the sum of three numbers thrown is
even, is a/b (given a & b are coprime) then
(A) a + b = 40 (B) a & b are both prime
(C) b – a = 13 (D) a + b = 41

7. Let f1(x) = sin–1(cos(sin2x)), f2(x) = cos–1(sin(cos2x)), f3(x) = sin–1(cos(cos2x)), f4(x) = cos–1(sin(sin2x)).


Then which of the following is correct
4 4
(A)  fi ( x )  2  2 (B)  ( 1)i fi (x)  0
i 1 i 1
4
(C)  ( 1)[i/3] fi (x)  0 where [.] denotes greatest integer function)
i 1

(D) f1(x) > f2(x) cos2x > 0

8. Through the vertex 'O' of the parabola y2 = 4ax, variable chords OP and OQ are drawn at right angles. If
the variable chord PQ intersects the axis of x at R, then distance OR :
(A) equals double the perpendicular distance of focus from the directrix.
(B) equals the semi latus rectum of the parabola
(C) equals latus rectum of the parabola
(D) equals double the latus rectum of the parabola

1 1 1
9. Find all positive integers x, y satisfying   .
x y 20

 h(r) .
1
10. Let f(x) = , g(x) = fofofofofofof(x) and h(x) = tan–1 (g(–x2 – x)), then find lim
1 x n
r 1

DPP No. # A23 (JEE–ADVANCED)


Total Marks : 31 Max. Time : 30 min.
Comprehension ('–1' negative marking) Q. 1 to Q.3 (3 marks 3 min.) [09, 09]
Single choice Objective ('-1' negative marking) Q. 4 to Q.6 (3 marks 3 min.) [09, 09]
Multiple choice objective ('-2' negative & Partial marking) Q.7 (4 marks 3 min.) [04, 03]
Subjective Questions ('–1' negative marking) Q.8 to Q.10 (3 marks 3 min.) [09, 09]

Question No. 1 2 3 4 5 6 7 8 9 10 Total


Marks Obtained

Comprehension # 1 (Q.1 to Q.3)


Let S denotes the set consisting of four function and S = {[x], sin–1 x, |x|, {x} }
where {x} denotes fractional part and [x] denotes greatest integer function. Let A, B, C are subsets of S.
Suppose
A : consists of odd (s)
B : consists of discontinuous function(s)
C : consists of non-decreasing function (s) or increasing function(s).
If f(x) A C ; g(x) B C ; h(x) B but not C and l(x) neither A nor B nor C.

1. The function l(x) is


(A) Periodic (B) Even (C) Odd (D) neither odd nore even

2. The range of g(f(x)) is


(A) {-1, 0, 1} (B) {-1, 0} (C) {0, 1} (D) {-2, -1, 0, 1}

Reg & Corp. Office : CG Tower, A-46 & 52, IPIA, Near City Mall, Jhalawar Road, Kota (Raj.)-324005
Website: www.resonance.ac.in | E-mail : contact@resonance.ac.in
PAGE NO.-32
Toll Free : 1800 258 5555 | CIN: U80302RJ2007PLC024029
DPPs BOOKLET-1 VIJETA (01-02JPA) | MATHEMATICS
3. The range of f(h(x)) is
   
(A)  0, (B) 0, (C)  0,  (D) 0, 
 2  2   2   2 

x 2
4. Number of solutions of the simultaneous equations |x| + |y| = 3 and sin = 1 is
2
(A) 4 (B) 6 (C) 10 (D) 12

1 1 1
5. The probability of hitting a target by three marksmen are , and respectively. The probability
2 3 4
that one and only one of them will hit the target when they fire simultaneously, is
11 1 1
(A) (B) (C) (D) None of these
24 12 8
3 5
6. Suppose A and B shoot independently until each hits his target. They have probabilities and of
5 7
hitting the targets at each shot respectively. The probability that B will require more shots than A is
6 7 8 1
(A) (B) (C) (D)
31 31 31 2

7. Let A be a variable point on the line y = 4. If B and C are variable points on the line y = – 4 such that
triangle ABC is equilateral, then which of the following statement(s) holds good?
(A) Perimeter of triangle ABC is 16 3
4
(B) Radius of the circle inscribed in triangle ABC is
3
(C) Locus of orthocentre of triangle ABC is 3y – 16 = 0
(D) r1 : r2 : r3 = 1 : 1 : 1, where r1 , r2 , r3 are ex-radii of triangle ABC.

Find the sum of all possible values of x satisfying arc cos  arc cos x  = arc sin  arc sin x  .
2 2
8.
     

9. If b > 0 find the number of value of ‘a’ for which domain and range of f(x) = ax 2  bx are equal.

10. Solve the following system of inequations : 4 (tan–1x)2 – 8 tan–1x + 3 < 0 & 4 cot–1x – (cot–1x)2 – 3  0

DPP No. # A24 (JEE–MAIN)


Total Marks : 60 Max. Time : 60 min.
Single choice Objective ('-1' negative marking) Q. 1 to Q.19 (3 marks 3 min.) [57, 57]
Subjective Questions ('–1' negative marking) Q.20 (3 marks 3 min.) [03, 03]

Ques. No. 1 2 3 4 5 6 7 8 9 10 11 12 13 14 15 16 17 18 19 20 Total


Mark
Obtained

1. A five digits number of the form x y z y x is choosen, probability that x < y is


35 6 19 13
(A) (B) (C) (D)
90 15 45 30
2. A pair of fair dice is thrown independently three times. The probability of getting a total of 9 exactly
twice is
1 8 8 8
(A) (B) (C) (D)
729 9 729 243

Reg & Corp. Office : CG Tower, A-46 & 52, IPIA, Near City Mall, Jhalawar Road, Kota (Raj.)-324005
Website: www.resonance.ac.in | E-mail : contact@resonance.ac.in
PAGE NO.-33
Toll Free : 1800 258 5555 | CIN: U80302RJ2007PLC024029
DPPs BOOKLET-1 VIJETA (01-02JPA) | MATHEMATICS
3. The probability of choosing randomly a number which is from 1 to 90 divisible by 6 or 8 is
1 11 1 23
(A) (B) (C) (D)
6 90 30 90

4. Which one of the following relations on R is equivalence relation-


(A) x R1y  |x| = |y| (B) x R2y  x  y
(C) x R3y  x | y (x divides y) (D) x R4y  x < y

5. Let U be the universal set and A B  C = U. Then {(A – B) (B – C) (C – A)} is equal to
(A) A B C (B) A (B C) (C) A B C (D) A (B C)

  
n  tan   ax  
 4 
6. lim , b  0 is equal to
x0 sinbx
a 2a a b
(A) (B) (C) (D)
b b 2b a

cos(sin(3x))  1
7. Lim equals
x0 x2
9 3 3 9
(A) (B) (C) (D)
2 2 2 2


8. If x = tan then 3x6 – 27x4 + 33x2 is equal to-
18
(A) 1 (B) 2 (C) 33 (D) 1/3

The function f(x) is defined for all real x, if f(x + y) = f 


xy 
9.  x , y and f(–4) = – 4, then f(2011) is
4   
(A) 2010 (B) 2012 (C) 4 (D) – 4

Let f(x) = 
sin x   2 sin 2x 
+..........+ 
10 sin10x 
10.  +   (where [y] is the greatest integer  y) .
 x   x   x 
then the value of Lim f(x) equals
x0
(A) 55 (B) 164 (C) 165 (D) 375

1  sin2x  cos2x
11. If f(x) = , then the value of f(16º) . f(29º) is
2cos2x
1 1 3
(A) (B) (C) 1 (D)
2 4 4

12. Range of the function f(x) = arc cot (2x – x2), is

 3    
(A)  , (B) 0 , (C) 0 , (D)  ,  

4 4    4   2  
4 

9x 2 sin2 x  4
13. The minimum value of the expression for x  (0, ) is
x sin x
16 8
(A) (B) 12 (C) 6 (D)
3 3

Reg & Corp. Office : CG Tower, A-46 & 52, IPIA, Near City Mall, Jhalawar Road, Kota (Raj.)-324005
Website: www.resonance.ac.in | E-mail : contact@resonance.ac.in
PAGE NO.-34
Toll Free : 1800 258 5555 | CIN: U80302RJ2007PLC024029
DPPs BOOKLET-1 VIJETA (01-02JPA) | MATHEMATICS
14. If (1 + sec2x). (1 + sec2y) = – x2 + 2x + 3, then
(A) x = – 1, y = n, n  (B) x = 0, y = n, n  (C) x = 1, y = 2n, n  (D) x = 1, y = n, n  

The range of the function f(x) = sin–1  x 2   + cos–1  x 2   , where [.] denotes the greatest integer
1 1
15.
 2  2
function, is
 
(A)  
(B) 
1
 ,  0,   (C) {} (D)  0, 
2   2  2 

4 1 4 1
16. If  = sin–1 + sin–1 and  = cos–1 + cos–1 then
5 3 5 3
(A)  <  (B)  =  (C)  >  (D) none of these

x b 
17. Evaluate lim , (where [ . ] denotes the greatest integer function).
x0 a  x 
b a
(A) (B) 0 (C) (D) does not exist
a b

 1 
 2  cot {x}  x
18. Lim   (where {.} and sgn(.) denotes fractional part function and signum function
x 0 sgn(x)  cos x
respectively) is equal to
(A) 2 (B) 1 (C) 0 (D) does not exist

1
19. Let f(x) = sin23x – cos22x and g(x) = 1 + tan–1 x , then the number of values of x in interval [–10,20]
2
satisfying the equation f(x) = sgn(g(x)), is
(A) 6 (B) 10 (C) 15 (D) 20

3
20. The probability of a shooter hitting a target is . How many minimum number of times must he fire so
4
that probability of hitting the target at least once is more than 0.99 ?

DPP No. # A25 (JEE–ADVANCED)


Total Marks : 32 Max. Time : 30 min.
Single choice Objective ('-1' negative marking) Q. 1 to Q.5 (3 marks 3 min.) [15, 15]
Multiple choice objective ('-2' negative & Partial marking) Q.6 to Q.7 (4 marks 3 min.) [08, 06]
Subjective Questions ('–1' negative marking) Q.8 to Q.10 (3 marks 3 min.) [09, 09]

Question No. 1 2 3 4 5 6 7 8 9 10 Total


Marks Obtained

1. A person throws dice, one the common cube and the other regular terahedron, the number on the
lowest face being taken in the case of a tetrahedron. The chance that the sum of numbers thrown is not
less than 5 is
1 3 4 5
(A) (B) (C) (D)
4 4 5 6

x(1 acos x)  b sin x


2. The values of a and b such that Lim = 1 are
x 0 x3
5 3 5 3 5 3 5 3
(A) , (B) ,– (C) – ,– (D) – ,
2 2 2 2 2 2 2 2
Reg & Corp. Office : CG Tower, A-46 & 52, IPIA, Near City Mall, Jhalawar Road, Kota (Raj.)-324005
Website: www.resonance.ac.in | E-mail : contact@resonance.ac.in
PAGE NO.-35
Toll Free : 1800 258 5555 | CIN: U80302RJ2007PLC024029
DPPs BOOKLET-1 VIJETA (01-02JPA) | MATHEMATICS

3. Lim 30  4 x  73 x equals
x 2  4x  7  3 6x  2
(A)15 (B) 2 (C) 3/2 (D) 0

 1 x 2  
If 3 sin–1 
2x 
 + 2 tan–1 
2x 
4. 2 
–4 cos–1  2 
= , where 0 < x < 1, then x is equal to
 1 x
2
 1 x    1 – x  3

1 1 3
(A) (B) – (C) 3 (D)
3 3 2

5. If in triangle ABC , A  (1, 10) , circumcentre    31 , 32  and orthocentre 


  113 , 34  then the

co-ordinates of mid-point of side opposite to A is :


(A) (1,  11/3) (B) (1, 5) (C) (1,  3) (D) (1, 6)

x 2  3x  5
6. If lim exists then k can be
x  4x  1  xk

(A) k = 2 (B) k < 2 (C) k > 2 (D) k  3

3x 2  ax  a  1
7. f (x) = then which of the following can be correct?
x2  x  2
(A) Lim f(x) exists  a=–2 (B) Lim f(x) exists  a = 13
x 1 x2

(C) Lim f(x) = 4/3 (D) Lim f(x) = – 1/3


x 1 x2

8. Three shots are fired independently at a target in succession. The probabilities of a hit in the first shot is
1 2 3
, in the second and in the third shot is . In case of one hit, the probability of destroying the
2 3 4
2 7
target is and in the case of two hits and in the case of three hits 1.0. Find the probability of
3 11
destroying the target in three shots.

9. There are 6 red balls and 6 green balls in a bag. Five balls are drawn out at random and placed in a red
box. The remaining seven balls are put in a green box. If the probability that the number of red balls in
p
the green box plus the number of green balls in the red box is not a prime number, is where p and q
q
are relatively prime, then find the value of (p + q)

x2 y2
10. Find the number of integral values of parameter 'a' for which three chords of the ellipse 2
+ =1
2a a2
 a2 
(other than its diameter) passing through the point P  11a,   are bisected by the parabola y2 = 4ax.
 4 

Reg & Corp. Office : CG Tower, A-46 & 52, IPIA, Near City Mall, Jhalawar Road, Kota (Raj.)-324005
Website: www.resonance.ac.in | E-mail : contact@resonance.ac.in
PAGE NO.-36
Toll Free : 1800 258 5555 | CIN: U80302RJ2007PLC024029
DPPs BOOKLET-1 VIJETA (01-02JPA) | MATHEMATICS

DPP No. # A26 (JEE–ADVANCED)


Revision DPP on "Binomial Theorem and Permutation and Combination"
Total Marks : 31 Max. Time : 30 min.
Multiple choice objective ('-2' negative & Partial marking) Q.1 (4 marks 3 min.) [04, 03]
Subjective Questions ('–1' negative marking) Q.2 to Q.10 (3 marks 3 min.) [27, 27]

Question No. 1 2 3 4 5 6 7 8 9 10 Total


Marks Obtained

10
 12 3 1 
 x  3x  4  12 
4
1. The coefficient of x160 in the expansion of (x8 + 1)60 is
 x x 
30 30
(A) C6 (B) C5 (C) divisible by 189 (D) divisible by 203

2. A insect initially at origin has to move from (0, 0) to (6, 6) . It moves towards +ve x-axis or+ve y-axis .In
each step it moves from (x, y) to (x + 1, y) or (x, y +1) .Find the number of different paths it can follow
to complete it’s journey if it does not pass through any of the four points ,(3,1) , (3,2) , (4,1) and (4,2).

3. In a chess tournament , ‘ n ’ players participate . Each player plays a game with every other participant
.All the players, except four juniors , score same points . Sum total of the score of four juniors is
17.5.Find ‘ n ’. ( Every game has a result or ends in a draw , in case of draw both players get 0.5 point
each , in case a winner is decided he gets one point & the loser gets zero ).

4. Number of ways in which eight digit number can be formed using the digits from 1 to 9 without
repetition, if first four places of the numbers are in increasing order and last 4 places are in decreasing
n
order, is 'n' then is equal to?
70

5. Five people of heights 65, 66, 67, 68 and 69 inches stand in a line. If they stand in an order that no
person can stand immediately before or after someone who is exactly 1 inch taller or exactly 1 inch
shorter than himself then find number of such arrangement?

1 1 1
6. If a, b, c are 3 natural numbers satisfying    1 then the number of possible ordered pairs
a b c
(a, b, c) is K, then K  1 is.

7. If n(A) = 7 and n(B) = 5, then the find the total number of surjective functions defined from set A to set
B.

8. Consider a grid with 8 parallel vertical edges and 9 parallel horizontal edges ,consecutive edges
being separated by distance unity-
(i) Find the number of rectangles in this grid.
(ii) Find the number of rectangles whose both sides are odd.
(iii) Find the number of squares with even sides.
(iv) Find the number of squares in this grid.
(v) Number of ways two squares of side , 1x1 , can be chosen so that they have a side in
common.

1 200
9. Prove that 100C0 100C2 + 100C2 100C4 + 100C4 100C6 +........+ 100C98 100C100 =  C98 100 C49  .
2
50
10. Find the coefficient of x25 in expansion of expression 
r 0
50
Cr (2x – 3)r (2 – x)50 – r.

Reg & Corp. Office : CG Tower, A-46 & 52, IPIA, Near City Mall, Jhalawar Road, Kota (Raj.)-324005
Website: www.resonance.ac.in | E-mail : contact@resonance.ac.in
PAGE NO.-37
Toll Free : 1800 258 5555 | CIN: U80302RJ2007PLC024029
DPPs BOOKLET-1 VIJETA (01-02JPA) | MATHEMATICS

DPP No. # A27 (JEE–ADVANCED)


Total Marks : 36 Max. Time : 30 min.
Single choice Objective ('-1' negative marking) Q. 1 to Q.4 (3 marks 3 min.) [12, 12]
Multiple choice objective ('-2' negative & Partial marking) Q.5 to Q.10 (4 marks 3 min.) [24, 18]

Question No. 1 2 3 4 5 6 7 8 9 10 Total


Marks Obtained

1. 
Let f(x) = max. | x2  2 | x ||, | x |   
and g(x) = min. | x2  2 | x ||, | x | , then
(A) both f(x) and g(x) are non differentiable at 5 points.
(B) f(x) is not differentiable at 5 points and g(x) is non differentiable at 7 points.
(C) number of points of non differentibility for f(x) and g(x) are 7 and 5 respectively.
(D) both f(x) and g(x) are non differentiable at 3 and 5 points respectively.

2. A draws a card from a pack of n cards marked 1, 2,.....,n. The card is replaced in the pack and B draws
a card.Then the probability that A draws a higher card than B is
n 1 1 n 1
(A) (B) (C) (D) None of these
2n 2 2n

8  9  11 
3. The value of cos–1  cos –1 
 + sin  sin
–1 
 + tan  tan  is equal to
 7   7   7 
5  11
(A) (B) (C) (D) 
7 7 7
4. The coordinates of a point P on the line 2x  y + 5 = 0, such that PA  PB is maximum,
where A is (4,  2) and B is (2,  4), will be :
(A) (11, 27) (B) ( 11,  17) (C) ( 11, 17) (D) (0, 5)

2h(x) | h(x) |
5. If g(x) = where h(x) = sin x – sinnx, n  R+, the set of positive real numbers, and
2h(x) | h(x) |
    
[g(x)], x   0,    , 
  2 2 
f(x) =  where [.] denotes greatest integer function. Then
 3, 
 x
2

(A) f(x) is continuous and differentiable at x = , when 0 < n < 1
2

(B) f(x) is continuous and differentiable at x = , when n > 1
2

(C) f(x) is discontinuous and non differentiable at x = , when 0 < n < 1
2

(D) f(x) is continuous but not differentiable at x = , when n > 1
2

6. If f(x) = max {sin x, sin–1 (cos x)}, then


(A) f is differentiable everywhere (B) f is continuous everywhere
n
(C) f is discontinuous at x = , n  (D) f is non-differentiable
2

7. Which of the following functions is periodic, where [ . ] denotes greatest integer function
(A) f(x) = 1[x] + (–1)[x] (B) g(x) = 1[5x] + (–1)[5x] (C) h(x) = 2[x] – (–2)[x] (D) (x) = 1[x] – (–1)[x]

Reg & Corp. Office : CG Tower, A-46 & 52, IPIA, Near City Mall, Jhalawar Road, Kota (Raj.)-324005
Website: www.resonance.ac.in | E-mail : contact@resonance.ac.in
PAGE NO.-38
Toll Free : 1800 258 5555 | CIN: U80302RJ2007PLC024029
DPPs BOOKLET-1 VIJETA (01-02JPA) | MATHEMATICS
8. Let f be a real valued function such that for any real x
f(15 + x) = f (15 – x) and f (30 + x) = – f (30 – x)
Then which of the following statements is NOT true ?
(A) f is odd and periodic (B) f is odd but not periodic
(C) f is even and periodic (D) f is even but not periodic

9. If f(x) = Lim x   [cos x]


n 2
3
 

n2  1  n2  3n  1  where [y] denotes largest integer  y, then identify

the correct statement (s).
3
(A) Lim f(x) = 0 (B) Lim f(x) =
x x
 4
2

3x 
(C) f(x) =  x  0,  (D) f(x) = 0  x    , 3 
2  2 2 2 

10. The integer ‘n’ for which


x3
cos2 x  cos x  e x cos x  e x 
l = lim 2 is a finite non-zero number, then:
x0 xn
1 1
(A) n = 2 (B) n = 4 (C) l = (D) l =
2 4

DPP No. # A28 (JEE–MAIN)


Total Marks : 60 Max. Time : 60 min.
Single choice Objective ('-1' negative marking) Q. 1 to Q.20 (3 marks 3 min.) [60, 60]

Ques. No. 1 2 3 4 5 6 7 8 9 10 11 12 13 14 15 16 17 18 19 20 Total


Mark
Obtained

1. Total number of equivalence relations defined in the set S = {a, b, c} is


(A) 5 (B) 3! (C) 23 (D) 33

2. Number of points of non-differentiability of the function


f(x) = [x2]{cos24x} + {x2}[cos24x] + x2sin24x + [x2][cos24x] + {x2}{cos24x} in (–50, 50) where [x] and {x}
denote the greatest integer function and fractional part function of x respectively, is equal to
(A) 98 (B) 99 (C) 100 (D) 0

(1  cos2x)(sec x  2tan x) 2
If f 
2 tan x 
3.  = then domain and range of f(x) are respectively :
 1  tan x 
2
2
(A) [0, 2] and [–1, 1] (B) [–1, 1] and [0, 2] (C) [–1, 1] and R (D) R and [0, 2]

4. A monic quadratic trinomial P(x) is such that P(x) = 0 and P(P(P(x))) = 0 have a common root, then
(A) P(0) · P(1) > 0 (B) P(0) · P(1) < 0 (C) P(0) · P(1) = 0 (D) none


5. Range of f(x)  cos   a sin x  cos x   , a  N , is [–1, 1], then amin is
 2 
(A) 1 (B) 2 (C) 3 (D) 4

4  x2
6. The function f(x) = is :
4x  x 3
(A) discontinuous at only one point (B) discontinuous at exactly two points
(C) discontinuous at exactly three points (D) None of these
Reg & Corp. Office : CG Tower, A-46 & 52, IPIA, Near City Mall, Jhalawar Road, Kota (Raj.)-324005
Website: www.resonance.ac.in | E-mail : contact@resonance.ac.in
PAGE NO.-39
Toll Free : 1800 258 5555 | CIN: U80302RJ2007PLC024029
DPPs BOOKLET-1 VIJETA (01-02JPA) | MATHEMATICS

7. The number of points where f (x) = [sin x + cos x], where [.] denotes the greatest integer function,
x  (0 , 2 ) is not continuous is :
(A) 3 (B) 4 (C) 5 (D) 6

8. If a,b and c are distinct real numbers such that a : (b + c) = b : (c + a) then


(A) a, b, c are all positive (B) a, b, c are all negative
(C) a + b + c = 0 (D) ab + bc + ca + 1 = 0

9. A set of ‘n’ numbers has the sum ‘s’. Each number of the set is increased by 20, then multiplied by
5 and then decreased by 20. The sum of the numbers in the new set thus obtained is :
(A) s + 20 n (B) 5s + 80n (C) s (D) 5s + 4n

1 1
10. If (21.4)a = (0.00214)b = 100, then the value of – is
a b
(A) rational which is not integral (B) prime
(C) irrational (D) composite

11. The number of positive integers 'n' such that n + 9, 16n + 9, 27n + 9 are all perfect squares is :
(A) 1 (B) 2 (C) 3 (D) 4

12. The solution set of the inequality 4 (cos–1x)2 – 1  0 is


  2 
(A)  1, cos  (B) 0,  (C) cos , 1 (D)  ,
1 1
 2   3   2  3 3 
 

13. If f(x) = cosec–1 (cosec x) and g(x) = cosec(cosec–1x) are equal functions then maximum range of
values of x is
     
(A)    
, 1  1 , (B)    
, 0   0 ,
 2   2   2   2 
(C) (– , –1]  [1, ) (D) [–1, 0)  [0, 1)

3
14. The number of solutions of the equation 2 sin–1 x 2  x  1 + cos–1 ( x2  x ) = is
2
(A) 0 (B) Infinite (C) 2 (D) 4

15. The mean and variance of 7 observations are 8 and 16 respectively. If 5 of the observations are
2, 4, 10, 12, 14. If the remaining two observations are x and y, then the value of xy is
(A) 48 (B) 42 (C) 32 (D) none of these

16. (p  ~ q)  (q  p) is logically equal to


(A) p  q (B) ~ p  q (C) p  ~q (D) ~(p  q)

n
17. Let Un =
n!
(n  2)!
where n  N. If Sn =
n 1

Un , then Lim Sn equals
n

1
(A) 2 (B)1 (C) (D) non existent
2

64  x2 x2 x2 x2 
18. lim  1  cos  cos  cos cos  is equal to
x 0 x 4 sin x 4  2 4 2 4 

1 1 1 1
(A) (B) (C) (D)
2 3 4 8

Reg & Corp. Office : CG Tower, A-46 & 52, IPIA, Near City Mall, Jhalawar Road, Kota (Raj.)-324005
Website: www.resonance.ac.in | E-mail : contact@resonance.ac.in
PAGE NO.-40
Toll Free : 1800 258 5555 | CIN: U80302RJ2007PLC024029
DPPs BOOKLET-1 VIJETA (01-02JPA) | MATHEMATICS
19. A pair of fair dice is rolled together till a sum of either 5 or 7 is obtained. Then the probability that
5 comes before 7, is
2 1 1 2
(A) (B) (C) (D)
5 5 3 3

20. If p : Ashok works hard ; q : Ashok gets good grade


The verbal form for (~p  q) is
(A) If Ashok works hard then he gets good grade
(B) If Ashok does not work hard then he gets good grade
(C) If Ashok does not work hard then he does not get good grade
(D) Ashok works hard if and only if he gets grade

DPP No. # A29 (JEE–ADVANCED)


Total Marks : 34 Max. Time : 30 min.
Single choice Objective ('-1' negative marking) Q. 1 to Q.2 (3 marks 3 min.) [06, 06]
Multiple choice objective ('-2' negative & Partial marking) Q.3 to Q.6 (4 marks 3 min.) [16, 12]
Subjective Questions ('–1' negative marking) Q.7 to Q.10 (3 marks 3 min.) [12, 12]

Question No. 1 2 3 4 5 6 7 8 9 10 Total


Marks Obtained

e|x|  e  x
1. Let f : R  R be a function defined by f(x) = . Then
ex  e x
(A) f is both one-one and onto (B) f is one-one but not onto
(C) f is onto but not one-one (D) f is neither one-one nor onto

2. (6, 0), (0, 6) and (7, 7) are the vertices of a triangle. The circle inscribed in the triangle has the equation
(A) x2 + y2  9x + 9y + 36 = 0 (B) x2 + y2  9x  9y + 36 = 0
(C) x2 + y2 + 9x  9y + 36 = 0 (D) x2 + y2  9x  9y  36 = 0

3. 
Function f(x) = Maximum sgn(x),  (9  x ),x is
2 3

(A) continuous everywhere (B) discontinuous at exactly one point
(C) non-differentiable at exactly 4 points (D) non-differentiable at exactly 5 points

4 If the lines L1 : 2x – 3y – 6 = 0, L2 : x + y – 4 = 0 and L3 : x + 2 = 0 taken pair wise in order constitute the


angles A, B and C respectively of ABC, then
15
(A)  tan A =
2
(B)  tan A tan B = 14
15
(C) tan A tan B tan C =
2
(D) equation whose roots are tan A, tan B , tan C is 2x3 – 15x2 + 28x – 15 = 0

1 1
5. Let a differentiable function f(x) be such that |f(x) – f(y)|  |x – y|  x, y  R and f(x)  . Then
2 2
the number of points of intersection of the graph of y = f(x) with
(A) the line y = x is one (B) the curve y = – x3 is one
(C) the curve 2y = |x| is three (D) None of these

Reg & Corp. Office : CG Tower, A-46 & 52, IPIA, Near City Mall, Jhalawar Road, Kota (Raj.)-324005
Website: www.resonance.ac.in | E-mail : contact@resonance.ac.in
PAGE NO.-41
Toll Free : 1800 258 5555 | CIN: U80302RJ2007PLC024029
DPPs BOOKLET-1 VIJETA (01-02JPA) | MATHEMATICS
6. A point P lies on or inside the circle x + y – 8x + 7 = 0 and ellipse 25x2 + 4y2 = 100 both. The point P
2 2

moves such that its path encloses the greatest possible area and is always at a fixed distance from
3 
point  ,0  in its region. If the locus of P is ax 2 + y2 + bx + c = 0 then which of the following is/are
2 
CORRECT ?
(A) a = 2 (B) b = –3 (C) c = 2 (D) a + b + c = 0

1 9 
7. Suppose that a parabola y = ax2 + bx + c, where a > 0 and (a + b + c) is an integer has vertex  , .
4 8 
p
If the minimum positive value of 'a' can be written as where p and q are relatively prime positive
q
integers, then find (p + q).

g(x) = log3 (x2 – 4x + b) ; h(x) = sin–1 


cx 
8. f : R  R, ƒ(x) = log2 (x3 + x2 + x + a) ; 
 1  x2 
 = number of values of 'a' for which of f(x) does not has range R
 = bmax for which range of g(x) is R.
 = cmax for which domain of h(x) is R.
Then () equals.

9. If the sum of first 16 terms of the series


 
1  2n
S = cot–1  22 
1
+ cot–1  23  2  –1  4
1 1  –1   , then
+ cot 2  + . . . . . . up to 16 terms is cot
 2   2



 3 
2   2 216 – 1
  
 
find the value of n.

10. Find all integral solution of x3 + 5y3 + 25z3 – 15xyz = 0.

DPP No. # A30(JEE–ADVANCED)


Total Marks : 36 Max. Time : 30 min.
Single choice Objective ('-1' negative marking) Q. 1 to Q.2 (3 marks 3 min.) [06, 06]
Multiple choice objective ('-2' negative & Partial marking) Q.3 to Q.8 (4 marks 3 min.) [24, 18]
Subjective Questions ('–1' negative marking) Q.9 to Q.10 (3 marks 3 min.) [06, 06]

Question No. 1 2 3 4 5 6 7 8 9 10 Total


Marks Obtained

1. Let ABC be a fixed triangle and P be variable point in the plane of triangle ABC. Suppose a, b, c are
lengths of sides BC, CA, AB opposite to angles A, B, C respectively. If a (PA) 2 +b (PB)2 + c(PC)2 is
minimum, then the point P with respect to ABC, is
(A) centroid (B) circumcentre (C) orthocentre (D) incentre

2. Two equal circles of radius R are touching each other externally . If a smaller circle of radius ' r ' is
touching both of these circles as well as their direct common tangent , then the ratio r : R is :
(A) 1 : 3 (B) 1 : 2 (C) 1 : 2 2 (D) 1 : 4

3. If 4a2 + c2 = b2 – 4ac, then the variable line ax + by + c = 0 always passes through two fixed points. The
coordinates of the fixed points can be
(A) (–2, –1) (B) (2, –1) (C) (–2, 1) (D) (2, 1)

Reg & Corp. Office : CG Tower, A-46 & 52, IPIA, Near City Mall, Jhalawar Road, Kota (Raj.)-324005
Website: www.resonance.ac.in | E-mail : contact@resonance.ac.in
PAGE NO.-42
Toll Free : 1800 258 5555 | CIN: U80302RJ2007PLC024029
DPPs BOOKLET-1 VIJETA (01-02JPA) | MATHEMATICS
 2 
 x cos if x0
4. Consider the function f(x) =  2x . Then which of the following is/are correct ?
 x0
 0 if
 1    1    1–    1–  
(A) f '    (B) f '    – (C) f '    – (D) f '   
3  2 3  2 3  2 3  2
       

5. Which of the following is/are correct ?


1
4  2x  x 2  4  2x  x 2 x  1  cos x  x
1
(B) lim 

(A) lim =– 2  = e 2
x0 2 x  2x x 0  x 
2
2
 ax  bx  c x x
(D) lim x 3  x 2  1  x 4  x 2  =
1
(C) lim   = (abc) 3
x 0 3 x   
  4 2

n  1  (k  1)(k  2)(k  1)k  120


6. If lim
n  cos
k 2



1
k(k  1)
=

 
, then

(A) number of even divisors of  is 24.


(B) sum of proper divisors of  is 1698.
(C) sum of proper divisors of  is 1697.
(D) number of ways in which  can be expressed as a product of two co prime factors is 4.

7. Let f(x) be defined as follows :


 
(cos x  sin x) ,  x0
cos ec x

 2
f(x) =  a , x0
 1/ x
 e e e 
2/ x 3/ x
, 0x
 ae2 / x  be3 / x 2
If f(x) is continuous at x = 0, then
1 1
(A) a  (B) a = e (C) b  (D) b = e
e e

8. Equation of a circle of radius 2 and touching the circles x 2 + y2 – 4| x | = 0 is


(A) x2 + y2 + 2 3 y + 2 = 0 (B) x2 + y2 + 4 3 y + 8 = 0
(C) x2 + y2 – 4 3 y + 8 = 0 (D) None of these
ax(x  1)  b ; x 1

9. Let f(x) =  x2 ; 1  x  3 is
 2
px  qx  2 ; x3
continuous for all x except x = 1 but | f(x) | is differentiable everywhere and f (x) is continuous at
k
x = 3 and | a + b + p + q | = , then find the value of k
18

10. Let the point M(2, 1) be shifted through a distance 3 2 units measured parallel to the line
L : x + y – 1 = 0 in the direction of decreasing ordinates, to reach at N. If the image of N in the line L is
R, then find the distance of R from the line 3x – 4y + 25 = 0.

Reg & Corp. Office : CG Tower, A-46 & 52, IPIA, Near City Mall, Jhalawar Road, Kota (Raj.)-324005
Website: www.resonance.ac.in | E-mail : contact@resonance.ac.in
PAGE NO.-43
Toll Free : 1800 258 5555 | CIN: U80302RJ2007PLC024029
DPPs BOOKLET-1 VIJETA (01-02JPA) | MATHEMATICS

DPP No. # A31 (JEE–ADVANCED)


Special DPP's on "Method of Differentiation"
Total Marks : 61 Max. Time : 60 min.
Single choice Objective (no negative marking) Q. 1 to 13 (3 marks, 3 min.) [39, 39]
Multiple choice objective (no negative marking) Q. 14 (4 marks, 3 min.) [04, 03]
Subjective Questions (no negative marking) Q.15 to 20 (3 marks, 3 min.) [18, 18]
Ques. No. 1 2 3 4 5 6 7 8 9 10 11 12 13 14 15 16 17 18 19 20 Total
Mark
Obtained

dy
1. If f(x) = 2x 2  1 and y = f(x2), then at x = 1 is equal to
dx
(A) 2 (B) 1 (C) – 2 (D) –1

2. If f(x) = logx (n x), then f  (x) at x = e is equal to


(A) 1/e (B) e (C) 1 (D) zero

dy dx
3. Let y = x3 – 8x + 7 and x = f(t). If = 2 and x = 3 at t = 0, then at t = 0 is given by
dt dt
19 2
(A) 1 (B) (C) (D) none of these
2 19

3
1 t 3 2  dy  dy
4. If x = ,y= + , then x    dx is equal to
t3 2t 2 t  
dx
(A) 0 (B) – 1 (C) 1 (D) 2

d2 y
5. If x = at2, y = 2at, then is equal to
dx 2
1 1 1 1
(A) – 2 (B) (C) – (D) –
t 2at 2 t3 2at3

dy
6. If x 1  y + y 1  x = 0, then is equal to
dx
1 1 1 1
(A) (B)  (C) (D)
(1 x) 2
(1 x) 2
1 x 2 (1  x)

x10
7. Let g is the inverse function of f and f  (x) = . If g(2) = a, then g  (2) is equal to
1  x 2

a 1  a2 a10 1  a10
(A) (B) (C) (D)
2 10
a10
1 a 2
a2

dy
8. If ax2 + 2 hxy + by2 = 0, then is equal to
dx
y x x
(A) (B) (D)  (D) None of these
x y y

Reg & Corp. Office : CG Tower, A-46 & 52, IPIA, Near City Mall, Jhalawar Road, Kota (Raj.)-324005
Website: www.resonance.ac.in | E-mail : contact@resonance.ac.in
PAGE NO.-44
Toll Free : 1800 258 5555 | CIN: U80302RJ2007PLC024029
DPPs BOOKLET-1 VIJETA (01-02JPA) | MATHEMATICS
 1  x2  x 4
d 
9. If   = ax + b, then the value of 'a' and 'b' are respectively
 1 x  x
2
dx 
(A) 2 and 1 (B)  2 and 1 (C) 2 and  1 (D) None of these

10. Let ef(x) = n x. If g(x) is the inverse function of f(x), then g (x) equals to:
x
(A) ex (B) ex + x (C) ex  e (D) ex + ln x

dy
11. If xm yn = (x + y)m + n, then is -
dx
xy x y
(A) (B) xy (C) (D)
xy y x

 1 dy
12. If 8 f(x) + 6 f   = x + 5 and y = x2 f(x), then at x = –1 is equal to
x dx
1 1
(A) 0 (B) (C) – (D) None of these
14 14

dy
13. If y = sec(tan–1x), then at x = 1 is equal to :
dx
1 1
(A) (B) (C) 1 (D) 2
2 2

dy
14. If 2x + 2y = 2x + y , then is equal to
dx

(A) 
2y
(B)
1
(C) 1  2y (D)

2x 1  2y 
2 x
1 2 x
2y 2 x
 1

15. Differentiate the following functions with respect to x .


5  x
(i) x2/3 + 7e – + 7 tan x (ii) x2. n x. ex (iii) n tan   
x 4 2
sin x  x cos x  1  cos x 
(iv) (v) tan  tan1  , 0 < x < 
x sin x  cos x  1  cos x
 

16. Differentiate the given functions w.r.t.x


(i) (n x)cos x (ii) xx – 2sinx (iii) y = (x n x)n n x

17. If f(x)  2 n (x  2)  x2 + 4x + 1, then find the solution set of the inequality f  (x)  0.

dy
18. Find if :
dx
 1 t
(i) x = a  cos t  n tan2  and y = a sin t.
 2 2
(ii) x = sint cos2t and y = cost cos2t

dy
19. Find , when x and y are connected by the following relations
dx

Reg & Corp. Office : CG Tower, A-46 & 52, IPIA, Near City Mall, Jhalawar Road, Kota (Raj.)-324005
Website: www.resonance.ac.in | E-mail : contact@resonance.ac.in
PAGE NO.-45
Toll Free : 1800 258 5555 | CIN: U80302RJ2007PLC024029
DPPs BOOKLET-1 VIJETA (01-02JPA) | MATHEMATICS

(i) ax2 + 2hxy + by2 +2gx + 2fy + c = 0 (ii) xy + xe–y + y . ex = x2


dy
20. Find in each of the following cases:
dx
4x 2 3x  1 x 2 
(i) y = tan–1 + tan–1 , (0 < x < 1) (ii) y = sin 1   , (0 < x < 1)
1  5x 2 3  2x  1 x 2
 
1 x
(iii) y = sin–1 , (– 1 < x < 1)
2

DPP No. # A32 (JEE–ADVANCED)


Special DPP's on "Method of Differentiation"
Total Marks : 64 Max. Time : 60 min.
Single choice Objective (no negative marking) Q.1 to 8 (3 marks, 3 min.) [24, 24]
Multiple choice objective (no negative marking) Q. 9 to 12 (4 marks, 3 min.) [16, 12]
Subjective Questions (no negative marking) Q.13 to 20 (3 marks, 3 min.) [24, 24]

Ques. No. 1 2 3 4 5 6 7 8 9 10 11 12 13 14 15 16 17 18 19 20 Total


Mark
obtained

1. Let f(x) be a polynomial in x. Then the second derivative of f(ex) w.r.t. x is
(A) f  (ex). ex + f  (ex) (B) f  (ex). e2x + f  (ex). e2x
x
(C) f  (e ) e 2x (D) f  (ex). e2x + f  (ex). ex
f g h
2. If f(x), g(x), h(x) are polynomials in x of degree 2 and F(x) = f  g h , then F(x) is equal to
f  g h
(A) 1 (B) 0 (C) –1 (D) f(x) . g(x) . h(x)

 d2 y   d2 x 
3. Statement - 1 Let f : [0, ) [0, ) be a function defined by y = f(x) = x2, then  2   2  = 1.
 dx 
   dy 
3
d2 y  dy 
d2 x
Statement - 2 –·  
dx 2 dy 2  dx 
(A) STATEMENT-1 is true, STATEMENT-2 is true and STATEMENT-2 is correct explanation for
STATEMENT-1
(B) STATEMENT-1 is true, STATEMENT-2 is true and STATEMENT-2 is not correct explanation for
STATEMENT-1
(C) STATEMENT-1 is true, STATEMENT-2 is false
(D) STATEMENT-1 is false, STATEMENT-2 is true
(E) Both STATEMENTS are false

4. Let y be an implicit function of x defined by x2x – 2xx cot y – 1 = 0. Then y(1) equals
(A) 1 (B) log 2 (C) – log 2 (D) – 1

5. Let f : (–1, 1)  R be a differentiable function with f(0) = – 1 and f(0) = 1. Let g(x) = [f(2f(x) + 2)]2.
Then g(0).
(A) – 4 (B) 0 (C) – 2 (D) 4

d2 x
6. =
dy 2

Reg & Corp. Office : CG Tower, A-46 & 52, IPIA, Near City Mall, Jhalawar Road, Kota (Raj.)-324005
Website: www.resonance.ac.in | E-mail : contact@resonance.ac.in
PAGE NO.-46
Toll Free : 1800 258 5555 | CIN: U80302RJ2007PLC024029
DPPs BOOKLET-1 VIJETA (01-02JPA) | MATHEMATICS
1 1 3
 d2 y   d2 y   dy   d2 y   dy 2  d2 y   dy 3
(A)  2  (B) –  2   dx  (C)  2   (D) –  2  
 dx   dx     dx   dx   dx   dx 
       
d2 y dy
7. If y = (sin–1 x)2 + (cos–1 x)2, then (1 – x2) 2
x is equal to
dx dx
(A) 4 (B) 3 (C) 1 (D) 0

 d   3 d2 y 
8. If y2 = P(x) is a polynomial of degree 3, then 2    y . 2  is equal to
 dx   dx 
(A) P  (x) + P  (x) (B) P  (x). P  (x) (C) P (x). P  (x) (D) a constant

t 1
9. The differential coefficient of sin-1 w.r.t. cos-1 is
1 t 2
1  t2
(A) 1  t > 0 (B) – 1  t < 0 (C) 1  t  R (D) none of these

f (x) d
10. If fn (x) = e n  1 for all n  N and fo (x) = x, then {fn (x)} is equal to
dx
d
(A) fn (x). {fn  1 (x)} (B) fn (x). fn  1 (x)
dx
(C) fn (x). fn  1 (x)........ f2 (x). f1 (x) (D) none of these

 
cos x  x 2  
sin x  x 2  cos x  x 2 
11. If f(x) = sin  x  x  2
cos  x  x 
2

sin x  x 2
 , then

sin 2x 0 sin 2x 2

(A) f(2) = 0 (B) f  (1/2) = 0 (C) f  (1) = – 2 (D) f  (0) = 4

12. If f is twice differentiable such that f(x) = –f(x) and f(x) = g(x). If h(x) is a twice differentiable function
such that h(x) = (f(x))2 + (g(x))2 . If h(0) = 2, h(1) = 4, then the equation y = h(x) represents
(A) a curve of degree 2 (B) a curve passing through the origin
(C) a straight line with slope 2 (D) a straight line with y intercept equal to 2.

x dy
13. If y = , then find
x dx
a
x
b
a  ...............

1  xn
14. It is known for x  1 that 1 + x + x2 +....+xn–1 = , hence find the sum of the series S = 1 + 2x +
1 x
3x2 ....+ (n + 1) xn.

.
..
..
ax dy y2 loge y
15. If y = a x , then prove that 
dx x(1  yloge x loge y)

16. Differentiate
1  2x   1  x2  1 
(i) tan 1   w.r.t. 1  4 x2 (ii) tan–1   with respect to tan–1(x)
 1 2 x   x 
 
Reg & Corp. Office : CG Tower, A-46 & 52, IPIA, Near City Mall, Jhalawar Road, Kota (Raj.)-324005
Website: www.resonance.ac.in | E-mail : contact@resonance.ac.in
PAGE NO.-47
Toll Free : 1800 258 5555 | CIN: U80302RJ2007PLC024029
DPPs BOOKLET-1 VIJETA (01-02JPA) | MATHEMATICS

2
d2 y  dy 
17. (i) If ey (x + 1) = 1, show that =   .
dx 2  dx 
d2 y dy
(ii) If y = sin (2 sin–1 x), show that (1 – x2) 2
=x – 4y.
dx dx

d2 y dy
18. If y = A e kt cos (p t + c), then prove that 2
+2k + n2 y = 0, where n2 = p2 + k2.
dt dt

19. If ex + y = xy, then show that


d2 y
=
 y (x 1)2  (y 1)2
.
 
dx 2 x2 (y 1)3

d2 y dy
20. If y = x n ((ax)–1 + a–1), prove that x (x + 1) 2
+x = y  1.
dx dx

DPP No. # A33 (JEE–ADVANCED)


Special DPP's on "Method of Differentiation"
Total Marks : 63 Max. Time : 60 min.
Single choice Objective (no negative marking) Q.1 to 10 (3 marks, 3 min.) [30, 30]
Multiple choice objective (no negative marking) Q.11 to 13 (4 marks, 3 min.) [12, 09]
Subjective Questions (no negative marking) Q. 14 to 20 (3 marks, 3 min.) [21, 21]

Ques. No. 1 2 3 4 5 6 7 8 9 10 11 12 13 14 15 16 17 18 19 20 Total


Mark
obtained

1. 
If y = sin1 x 1  x  x 1 x 2  and dydx = 2 1
x(1  x)
+ p, then p is equal to

1 1
(A) 0 (B) (C) sin1 x (D)
1 x 1  x2
dn
2. If u = ax + b, then (f(ax + b)) is equal to
dx n
dn dn dn dn
(A) (f(u)) (B) a (f(u)) (C) an n (f(u)) (D) an (f(u))
dun dun du dx n

d2 x
3. If y = x + ex , then is equal to
dy 2
ex ex 1
(A) ex (B)  (C)  (D)
1 e  1 e  1 e 
3 2 2
x x x

4. If y is a function of x and n (x + y) – 2xy = 0, then the value of y(0) is equal to


(A) 1 (B) –1 (C) 2 (D) 0

5. Let S denote the set of all polynomials P(x) of degree  2 such that P(1) = 1, P(0) = 0 and
P(x) > 0 x  [0, 1], then
(A) S =  (B) S = {(1 – a)x2 + ax ; 0 < a < 2}

Reg & Corp. Office : CG Tower, A-46 & 52, IPIA, Near City Mall, Jhalawar Road, Kota (Raj.)-324005
Website: www.resonance.ac.in | E-mail : contact@resonance.ac.in
PAGE NO.-48
Toll Free : 1800 258 5555 | CIN: U80302RJ2007PLC024029
DPPs BOOKLET-1 VIJETA (01-02JPA) | MATHEMATICS

(C) S = {(1 – a) x2 + ax ; 0 < a < 1} (D) S = {(1 – a) x2 + ax ; 0 < a < }

6. If x cos y + y cos x =  then the value of y (0) is equal to


(A)  (B) –  (C) 1 (D) 0

2 2
  x    x 
7. If f(x) = – f(x) and g(x) = f(x) and F(x) =  f    +  g    and given that F(5) = 5, then F(10) is
  2    2 
equal to
(A) 5 (B) 10 (C) 0 (D) 15

8. Let g(x) = log f(x), where f(x) is a twice differentiable positive function on (0, ) such that
 1  1
f(x + 1) = x f(x). Then, g  N   – g   is equal to, for N = 1, 2, 3, .......
 2  2
 1 1
 1 
  1 1
 1 

(A) – 4 1    ....   (B) 4 1    ....  

 9 25 (2N  1) 
2
 
 9 25 (2N  1) 
2


 1 1 1 
 
 1 1 1 

(C) – 4 1    ....   (D) 4 1    ....  

 9 25 (2N  1)2

 
 9 25 (2N  1)2

9. The area of the closed figure bounded by y = x, y =  x and the tangent to the curve y = x2  5 at the
point (3, 2) is
15 35
(A) 5 (B) (C) 10 (D)
2 2

 1  6x x 
10. If for x   0,  , the derivative of tan–1   is x . g(x), then g(x) equals :
 4  1  9x
3

9 3x x 3x 3
(A) (B) (C) (D)
1  9x3 1  9x 3 1  9x3 1  9x3

 y 
11. If x2  y2 = et where t = sin–1   , then dy is equal to
 x2  y2  dx
 
xy xy yx xy
(A) (B) (C) (D)
xy xy yx 2x  y

12. Let f(x) = x sin x, x > 0. Then for all natural numbers n, f (x) vanishes at
 1  1 
(A) a unique point in the interval  n, n   (B) a unique point in the interval  n  ,n  1
 2  2 
(C) a unique point in the interval (n, n +1) (D) two points in the interval (n, n +1)

13. The functions u = ex sin x; v = ex cos x satisfy the equation


du dv d2 u
(A) v u = u2 + v2 (B) =2v
dx dx dx 2
d2 v du dv
(C) 2
=2u (D) + = 2v
dx dx dx

Reg & Corp. Office : CG Tower, A-46 & 52, IPIA, Near City Mall, Jhalawar Road, Kota (Raj.)-324005
Website: www.resonance.ac.in | E-mail : contact@resonance.ac.in
PAGE NO.-49
Toll Free : 1800 258 5555 | CIN: U80302RJ2007PLC024029
DPPs BOOKLET-1 VIJETA (01-02JPA) | MATHEMATICS
dPn
14. If Pn is the sum of a GP upto n terms. Show that (1 – r) = n . Pn–1 – (n – 1) Pn.
dr

x x x sin x 1 x 1 x 1
15. If cos . cos 2 . cos 3 ....  = , then find the value of 2
sec2 + 4 sec2 2
+
2 2 2 x 2 2 2 2 26
x
sec2 3 + ...... 
2

 x d2 y dy
16. Show that the substitution z = n  tan  changes the equation + cot x + 4y cosec2x = 0 to
 2  dx 2
dx
(d2y/dz2) + 4y = 0.

F" f " g" 2c F f  g


17. If F(x) = f(x). g(x) and f (x). g (x) = c, prove that    and  
F f g fg F f g

ax 2 bx c y' 1  a b c 
18. If y =    
x  a  x b  x c  x 
+ + +1, then prove that
(x  a)(x  b)(x  c) (x  b)(x  c) x  c y

2
19.
 dy 
If x = sec  – cos  and y = secn – cosn  then show that x2  4 
 
dx

  n (y  4)
2 2

y 2
d2 y  dy 
20. If (a + bx) e x = x, then prove that x3 2
= x  y
dx  dx 

DPP No. # A34 (JEE–ADVANCED)


Total Marks : 35 Max. Time : 30 min.
Single choice Objective ('-1' negative marking) Q. 1 to Q.2 (3 marks 3 min.) [06, 06]
Multiple choice objective ('-2' negative & Partial marking) Q.3 to Q.7 (4 marks 3 min.) [20, 15]
Subjective Questions ('–1' negative marking) Q.8 to Q.10 (3 marks 3 min.) [09, 09]

Question No. 1 2 3 4 5 6 7 8 9 10 Total


Marks Obtained

1. The internal common tangents to two circles with centre C1 and C2 intersect the line joining C1 and C2 at
P and the two direct common tangents intersects the line joining C 1 and C2 at Q. The lengths C1P, C1C2
and C1Q are in
(A) A.P. (B) GP (C) HP (D) None of these

2. An object is moving in the clockwise direction around the unit circle x 2 + y2 = 1 . As it passes through
the point (1/2, 3 / 2 ), its y-coordinate is decreasing at the rate of 3.units per second .The rate at which
the x-coordinate changes at this point is (in units per second)
(A) 2 (B) 3 3 (C) 3 (D) 2 3

3. Let P and Q be any two points on the lines represented by 2x – 3y = 0 and 2x + 3y = 0 respectively. If
the area of triangle OPQ (where O is origin) is 5, then the equation of the locus of mid-point of PQ, can
be equal to
(A) 4x2 – 9y2 + 30 = 0 (B) 9x2 – 4y2 + 30 = 0
(C) 9x – 4y – 30 = 0
2 2 (D) 4x2 – 9y2 – 30 = 0

Reg & Corp. Office : CG Tower, A-46 & 52, IPIA, Near City Mall, Jhalawar Road, Kota (Raj.)-324005
Website: www.resonance.ac.in | E-mail : contact@resonance.ac.in
PAGE NO.-50
Toll Free : 1800 258 5555 | CIN: U80302RJ2007PLC024029
DPPs BOOKLET-1 VIJETA (01-02JPA) | MATHEMATICS
3
4. The area of a triangle is square units. Two of its vertices are the points A (2, –3) and B(3, –2), the
2
centroid of the triangle lies on the line 3x – y – 2 = 0, then third vertex C is
(A) (–8, – 10) (B) (–8, 10) (C) (–11, – 19) (D) (11, – 19)

 Min f(t) : 0  t  x ; 0  x  1
5. If f(x) = 4x3  x2  2x + 1 and g(x) =  then
3  x ; 1 x  2

1 5
(A) g 
1
(B) g 
1
+ g   + g   =
3 5
 = 
4 2 4 4 4 4
5 7
(C) g 
1
+ g   + g   = (D) g 
1
+ g   + g   =
3 5 3 5
 
4 4 4 2 4 4 4 2

6. Consider the equation, sin4x – cos2 x sin x + 2 sin2 x + sin x = 0 in 0  x  3
(A) Number of solution of the equation is 4
(B) Sum of the solution of the equation is 6 
(C) Product of the solution of the equation is 0
(D) 2 is not the solution

7. f(x) = cos–1(cosx) + sin–1(sinx), then


f(x)
(A) lim 2
x 0 x
f(x)
(B) lim does not exists
x 0 x
2
(C) Area of the triangle formed by y = f(x) & y = x is
8
2
(D) Area of the triangle formed by y = f(x) & y = x is
4

min f(t); 0  t  x, 0  x  6
8. Let f(x) = x3 – 9x2 + 15x + 6 and g(x) = 
 x  18 ; x6
Draw the graph of g(x) and discuss the continuity and differentiability of g(x).

9. If ,  are two distinct real roots of the equation ax 3 + x – 1 – a = 0, (a  – 1, 0), none of which is equal to
a (k  )
unity, then the value of lim (1  a)x  x  a is
3 2
. Find the value of k.
x
1 (e1x  1)(x  1) 


10. Circles A and B are externally tangent to each other and to line t. The sum of the radii of the two circles
is 12 and the radius of circle A is 3 times that of circle B. The area in between the two circles and this
b
external tangent, 't', is a 3 – then find the value of a + b.
2

DPP No. # A35 (JEE–MAIN)


Total Marks : 60 Max. Time : 60 min.
Single choice Objective ('-1' negative marking) Q. 1 to Q.20 (3 marks 3 min.) [60, 60]

Ques. No. 1 2 3 4 5 6 7 8 9 10 11 12 13 14 15 16 17 18 19 20 Total


Mark
Obtained

Reg & Corp. Office : CG Tower, A-46 & 52, IPIA, Near City Mall, Jhalawar Road, Kota (Raj.)-324005
Website: www.resonance.ac.in | E-mail : contact@resonance.ac.in
PAGE NO.-51
Toll Free : 1800 258 5555 | CIN: U80302RJ2007PLC024029
DPPs BOOKLET-1 VIJETA (01-02JPA) | MATHEMATICS

1. In ABC, angle A is 120°, BC + CA = 20 and AB + BC = 21, then the length of the side BC, equals
(A) 13 (B) 15 (C) 17 (D) 19

 3 5 7
2. Value of cos4 + cos4 + cos4 + cos4 is
8 8 8 8
1 3
(A) (B) (C) 1 (D) 0
2 2

3. Range of the function f(x) = sin1 | sinx |  cos1 | cos x | is

(A) {0} (B)  0, / 2  (C)  0,   (D) None of these


   

x 1 a
4. If f(x) = and f(f(1)) = in lowest form a, b N then
x 1 2 b
(A) a + b = 2 (B) a – b = 0 (C) ab = 1 (D) all of these

5. The set of values of a for which x2 + ax + sin–1 (x2 – 4x + 5) + cos–1 (x2 – 4x + 5) = 0 has at least one
solution is
(A) ( – , – 2 ]  [ 2 , ] (B) ( – , – 2 )  ( 2 , )
(C) R (D) a = –(2 + /4)

6. If f (x) + f (y) + f (xy) = 2 + f (x) . f (y) , for all real values of x & y and f (x) is a polynomial function with
f (4) = 17 , then find the value of f (5) .
(A) 26 (B) 25 (C) 21 (D) 27

7. Consider a function f(x) : R R and if Lim [f(x)] does not exist, where [ ] denotes greatest integer
xa
function, then

(A) Lim f(x) will never exist (B) f(x) may be continuous at x = a
xa
(C) Function will not have a tangent at x = a (D) None of these

1/ x
 f (1  x) 
8. Let f: R  R be such that f (1) = 3 & f  (1) = 6. Then Limit   is equal to
x  0  f (1) 

(A) 1 (B) e1/2 (C) e2 (D) e3

n
 
The value of lim  a  1  b  ; a, b,n  N is equal to :
n
9.
 n  a
 
(A) a
b (B) b a (C) b (D) a 

(sin(x  h))ln(x h)  (sin x)ln x 


10. Let f(x) = Lim then f   is
h0 h 2

(A) equal to 0 (B) equal to 1 (C) ln (D) non existent
2
11. If ax2 + bx + c = 0 has imaginary roots and a – b + c > 0, then the set of points (x, y) satisfying the
equation
 y
a  x2   (b  1) x  c = |ax2 + bx + c| + | x + y |
 a
Reg & Corp. Office : CG Tower, A-46 & 52, IPIA, Near City Mall, Jhalawar Road, Kota (Raj.)-324005
Website: www.resonance.ac.in | E-mail : contact@resonance.ac.in
PAGE NO.-52
Toll Free : 1800 258 5555 | CIN: U80302RJ2007PLC024029
DPPs BOOKLET-1 VIJETA (01-02JPA) | MATHEMATICS
consists of the region in the xy-plane which is
(A) on or above the bisector of I and III quadrant (B) on or above the bisector of II and IV quadrant
(C) on or below the bisector of I and III quadrant (D) on or below the bisector of II and IV quadrant

12. The sets {(x, y) : | y – 1| – x  1}, {(x, y) : | x | – y  1}


{(x, y) : x – | y |  1} {(x, y) : y – | x – 1 |  0}
are represented by the shaded regions in the figures given below in some order

F2

F1

(1, 0) (2, 1)
(–1, 0) (0, 1)
(0, –1) (1, 0)

F3 F4
(0, 2)
(–1, 1)
(2, 1)

(1, 0)
(2, –1)

Then the correct order of the figures is-

(A) F4, F1, F2, F3 (B) F4, F2, F3, F1 (C) F1, F4, F3, F2 (D) F4, F1, F3, F2

n  2
13. If cos–1  > , then the minimum and maximum values, of integer n are respectively
  3
2
(A) – 6 and – 3 (B) – 6 and – 4 (C) 3 and 6 (D) 4 and 6

14. If L1 & L2 are the lengths of the segments of any focal chord of the parabola y2 = x, then
1 1 1 1 1 1 1 1 1 1
(A) + =2 (B) + = (C) + =4 (D) + =
L1 L2 L1 L2 2 L1 L2 L1 L2 4

x2 y2 x2 y2
15. Area of the quadrilateral formed with the foci of the hyperbola 2
 2
 1 and 2
  1 is
a b a b2
1 2
(A) 4(a2 + b2) (B) 2(a2 + b2) (C) (a2 + b2) (D) (a + b2)
2

16. A parabola y = ax2 + bx + c crosses the x  axis at (, 0) (, 0) both to the right of the origin. A circle also
passes through these two points. The length of a tangent from the origin to the circle is :
bc b c
(A) (B) ac2 (C) (D)
a a a

ax  b
17. If S.D. of a variable x is  then S.D. of is =
c

Reg & Corp. Office : CG Tower, A-46 & 52, IPIA, Near City Mall, Jhalawar Road, Kota (Raj.)-324005
Website: www.resonance.ac.in | E-mail : contact@resonance.ac.in
PAGE NO.-53
Toll Free : 1800 258 5555 | CIN: U80302RJ2007PLC024029
DPPs BOOKLET-1 VIJETA (01-02JPA) | MATHEMATICS

a a 
(A)   b  (C)   | b |
a a
(B)  (D) 
c  c c  c

18. Two points in a plane are related If OA = OB, where O is a fixed point this relation is :
(A) Reflexive but not symmetric (B) Reflexive but not transitive
(C) Transitive but not symmetric (D) Equivalence relation

19. The relation ''less than'' in the set of natural number is


(A) Only symmetric (B) Only transitive (C) Only reflexive (D) Equivalence relation

20. Area of the triangle formed by the tangents at the points (4, 6), (10, 8) and (2, 4) on the parabola
y2 – 2x = 8y – 20, is (in sq. units)
(A) 4 (B) 2 (C) 1 (D) 8

DPP No. # A36 (JEE–ADVANCED)


Revision DPP on "Straight Line"
Total Marks : 32 Max. Time : 30 min.
Single choice Objective ('-1' negative marking) Q. 1 to Q.4 (3 marks 3 min.) [12, 12]
Multiple choice objective ('-2' negative & Partial marking) Q.5 to Q.6 (4 marks 3 min.) [08, 06]
Subjective Questions ('–1' negative marking) Q.7 to Q.10 (3 marks 3 min.) [12, 12]

Question No. 1 2 3 4 5 6 7 8 9 10 Total


Marks Obtained

1. A variable line is drawn through origin O to cut two fixed straight lines L 1 : y – c = 0 and L2 : ax + by = 1
mn m n
in R and S. A point P is chosen on the variable line such that = + . The locus of P
OP OR OS
which is
(A) cn (ax + by – 1) + m(y – c) = 0 (B) n (ax + by – 1) + m(y – c) = 0
(C) cn (ax + by – 1) + (y – c) = 0 (D) n (ax + by – 1) + (y – c) = 0

2. Chords of the curve 4x2 + y2 – x + 4y = 0 which subtend a right angle at the origin pass through a fixed
point whose co-ordinates are :
1 4  1 4 1 4  1 4
(A)  ,   (B)   ,  (C)  ,  (D)   , 
5 5  5 5 5 5  5 5

3. The base BC of a  ABC is bisected at the point (p, q) & the equation to the side AB & AC are
px + qy = 1 & qx + py = 1. The equation of the median through A is :
(A) (p  2q) x + (q  2p) y + 1 = 0
(B) (p + q) (x + y)  2 = 0
(C) (2pq  1) (px + qy  1) = (p2 + q2  1) (qx + py  1)
(D) None of these
Orthocentre of an acute triangle ABC is at the origin and its circumcentre has the co-ordinates  , 
1 1
4.
2 2 
. If the base BC has the equation 4x – 2y = 5, then the radius of the circle circumscribing the triangle
ABC, is
5 3
(A) (B) 3 (C) (D) 6
2 2
5. A variable line cuts the lines x2 – (a + b) x + ab = 0 in such a way that intercept between the lines
subtends a right angle at origin. Then
(A) locus of the foot of the perpendicular from origin on the variable line is x 2 + y2 – (a + b)x + ab = 0
(B) locus of the foot of the perpendicular from origin on the variable line is straight line
(C) locus of the foot of the perpendicular from origin on the variable line is circle

Reg & Corp. Office : CG Tower, A-46 & 52, IPIA, Near City Mall, Jhalawar Road, Kota (Raj.)-324005
Website: www.resonance.ac.in | E-mail : contact@resonance.ac.in
PAGE NO.-54
Toll Free : 1800 258 5555 | CIN: U80302RJ2007PLC024029
DPPs BOOKLET-1 VIJETA (01-02JPA) | MATHEMATICS
(D) locus of the foot of the perpendicular from origin on the variable line is (a + b)x + ab = 0

6. For all values of , the lines represented by the equation


(2 cos + 3 sin ) x + (3 cos – 5 sin ) y – (5 cos – 2 sin ) = 0
(A) pass through a fixed point
(B) vertex of the system is (1, 1)
5
(C) pass through the origin if tan  =
2
(D) the line 3x – 4y = 3 is one of the member of the family
7. The distance between the two parallel lines is 1 unit . A point 'A' is chosen to lie between the lines at a
distance 'd' from one of them . Triangle ABC is equilateral with B on one line and C on the other parallel
line . The length of the side of the equilateral triangle is ______ .

8. A line cuts the x-axis at A (7, 0) and the y-axis at B(0,– 5). A variable line PQ is drawn perpendicular to
AB cutting the x-aixs in P and the y-axis in Q. If AQ and BP intersect at R, find the locus of R.

9. A rectangle PQRS has its side PQ parallel to the line y = mx & vertices P, Q & S on the lines y = a, x = b
& x = b, respectively.Find the locus of the vertex R.

10. Let A(0, 1), B(1, 1), C(1, –1), D(–1, 0) be four points. If P be any other point, then
PA + PB + PC + PD  d. Then find [d] (where [.] represents greatest integer)

DPP No. # A37 (JEE–ADVANCED)


Total Marks : 36 Max. Time : 30 min.
Single choice Objective ('-1' negative marking) Q. 1 to Q.3 (3 marks 3 min.) [09, 09]
Multiple choice objective ('-2' negative & Partial marking) Q.4 to Q.9 (4 marks 3 min.) [24, 18]
Subjective Questions ('–1' negative marking) Q.10 (3 marks 3 min.) [03, 03]

Question No. 1 2 3 4 5 6 7 8 9 10 Total


Marks Obtained

       
The range of f(x) = cos  sin  n  x  e    + sin  cos  n  x  e    is
2 2
1.
   x2  1       x2  1   
       
(A) [cos (sin 1), sin (cos 1)] (B) [sin (cos 1) + 1, cos (sin 1)]
(C) [cos (sin 1) + sin (cos 1), 1 + sin 1) (D) (0, 1)

2. The area of the quadrilateral formed by the tangents at the end points of latus rectum of the ellipse
x2 y2
+ = 1 is (in sq. units)
9 5
(A) 27/4 (B) 9 (C) 27/2 (D) 27

3. The diagram shows the dimensions of the floor of an L-shaped room. (All the angles are right angles).
The area of the largest circle that can be drawn on the floor of this room, is

81 145
(A) 16 (B) 25  (C) (D)
4 4

Reg & Corp. Office : CG Tower, A-46 & 52, IPIA, Near City Mall, Jhalawar Road, Kota (Raj.)-324005
Website: www.resonance.ac.in | E-mail : contact@resonance.ac.in
PAGE NO.-55
Toll Free : 1800 258 5555 | CIN: U80302RJ2007PLC024029
DPPs BOOKLET-1 VIJETA (01-02JPA) | MATHEMATICS
4. The ordered pairs (x, y) of real number satisfying 4x2 – 4x + 2 = sin2y and x2 + y2  3. is equal to
1  1  1  1 
(A)  ,  (B)  ,  (C)  ,  (D)  , 
2 2 2 4 2 2  2 4 
5. For the straight lines 4x + 3y – 7 = 0 and 24x + 7y – 31 = 0, the equation of
(A) bisector of the obtuse angle between them is x – 2y + 1 = 0
(B) bisector of the acute angle between them is 2x + y – 3 = 0
(C) bisector of the angle containing origin is x – 2y + 1 = 0
(D) bisector of the angle containing the point (1, – 2) is x – 2y + 1 = 0

m n
6. The equation of straight line with gradient 1, passing through  ,  where m, n R, satisfies the
 2 2
   
equation sec2 (n (m + 2)) + m2 = 1  where n   ,   , can be
  2 2
 
(A) x + y = 0 (B) x – y = 0 (C) x – y + =0 (D) x – y  =0
4 4

7. The solution set of the equality |x2 + 8x + 7| = |x2 + 4x + 4| + |4x + 3| for x  R are
3   3 
(A) (– 2, ) (B)  ,    {– 2} (C)   ,    {– 2} (D) [0, )
4   4 
8. All the values of 'a' for which the quadratic expression ax 2 + (a – 2) x – 2 is negative for exactly two
integral values of x may lie in
(A) [1, 3/2) (B) [3/2, 2) (C) [1, 2) (D) [–1, 2)

9. The radius of a right circular cyliner increases at a constant rate. Its altitude is a linear function of the
radius and increases three times as fast as radius. When the radius is 1 cm the altitude is 6 cm. When
the radius is 6cm, the volume is increasing at the rate of 1 Cu cm/sec. When the radius is 36cm, the
volume is increasing at a rate of n cu. cm/sec. The value of 'n' is divisible by :
(A) 11 (B) 22 (C) 3 (D) 33

x2 y2
10. If the ellipse  =1(a >b) and parabola y2 = 4cx(c > 0) have common latus rectum, then the value
a2 b2
of   is (where [.] deonotes greatest integer function.
a
 
c

DPP No. # A38 (JEE–ADVANCED)


Total Marks : 39 Max. Time : 35 min.
Single choice Objective ('-1' negative marking) Q. 1 to Q.4 (3 marks 3 min.) [12, 12]
Multiple choice objective ('-2' negative & Partial marking) Q.5 to Q.8 (4 marks 3 min.) [16, 12]
Subjective Questions ('–1' negative marking) Q.9 (3 marks 3 min.) [03, 03]
Match the Following (each row '(2, -1)' negative marking) Q.10 (8 marks, 8 min.) [08, 08]

Question No. 1 2 3 4 5 6 7 8 9 10 Total


Marks Obtained

1. Which of the following expresses the circumference of a circle inscribed in a sector OAB with radius R
and AB=2a?

Reg & Corp. Office : CG Tower, A-46 & 52, IPIA, Near City Mall, Jhalawar Road, Kota (Raj.)-324005
Website: www.resonance.ac.in | E-mail : contact@resonance.ac.in
PAGE NO.-56
Toll Free : 1800 258 5555 | CIN: U80302RJ2007PLC024029
DPPs BOOKLET-1 VIJETA (01-02JPA) | MATHEMATICS
Ra 2R 2 R
(A) 2 (B) (C) 2(R – a)2 (D) 2
Ra 9 Ra

2. If the straight lines joining the origin and the points of intersection of the curve
5x2 + 12xy  6y2 + 4x  2y + 3 = 0 and x + ky  1 = 0
are equally inclined to the x- axes then the value of k :
(A) is equal to 1 (B) is equal to  1
(C) is equal to 2 (D) does not exist in the set of real numbers .

nx
3. Range of the function f(x) = is
x
(C)  ,
2
(D)  ,
1
(A) (– , e) (B) (– , e2)
 e   e 

4. The tangent to the hyperbola xy = c2 at the point P  ct, c  intersects the x-axis at T and the y-axis at
 t
T. The normal to the hyperbola at P intersects the x-axis at N and the y-axis at N. The areas of the
1 1
triangles PNT and PN'T' are  and ' respectively, then  is
 '
(A) equal to 1 (B) depends on t (C) depends on c (D) equal to 2

5. Function f(x)= sin (n x) – cos(lnx) is


 2n

2n
3   2n
3
2n
7 
(A) M.I. in e 4, e 4  , n  (B) M.D. in e 4 , e 4  , n 
   
 2n
5
2n
   2n
3
2n
7 
(C) M.D. in e 4 , e 4  , n  (D) M.I. in e 4 , e 4  , n 
   

6. If the lines joining the points of intersection of the curve 4x 2 + 9y + 18xy = 1 and the line y = 2x + c to
the origin are equally inclined to the y-axis, then c is :
1 9  161 1 2
(A) (B) (C) 1,  (D)
3 8 2 3
2  2x  1 
7. If f(x) = tan–1   – n (x2 + x + 1) + (k2 – 5k + 3) x + 10 is a decreasing function for all x  R,
3  3 
then possible value of k is
3 7
(A) (B) 2 (C) 3 (D)
4 2
8. If f(x) = x3 + bx2 + cx + d and 0 < b2 < c, then which of following is INCORRECT?
(A) f(x) is a strictly increasing function (B) f(x) has local maxima
(C) f(x) is a strictly decreasing function (D) f(x) is bounded

9. Two men P and Q start with velocities v at the same time from the junction of two roads inclined at 45º
to each other. If they travel by different roads, find the rate at which they are being separated.

10. Match the columns :


Column- I Column-II
5
(A) f(x) =  (x + r), then f' (– 5) is (P) 1
r 1
x2  x  1 dy
(B) y= , then  0 in [– , ] where 2 is (Q) 2
x2  x  1 dx

Reg & Corp. Office : CG Tower, A-46 & 52, IPIA, Near City Mall, Jhalawar Road, Kota (Raj.)-324005
Website: www.resonance.ac.in | E-mail : contact@resonance.ac.in
PAGE NO.-57
Toll Free : 1800 258 5555 | CIN: U80302RJ2007PLC024029
DPPs BOOKLET-1 VIJETA (01-02JPA) | MATHEMATICS

d2 y
(C) x3 + 3x2y + xy2 – 5y3 = 0 then is (R) 0
dx2 (1, 1)

(D) x3 + y3 = 8, Number of straight lines through origin


which do not meet this curve is (S) 24

DPP No. # A39 (JEE–ADVANCED)


Total Marks : 36 Max. Time : 30 min.
Single choice Objective ('-1' negative marking) Q. 1 to Q.3 (3 marks 3 min.) [09, 09]
Multiple choice objective ('-2' negative & Partial marking) Q.4 to Q.9 (4 marks 3 min.) [24, 18]
Subjective Questions ('–1' negative marking) Q.10 (3 marks 3 min.) [03, 03]

Question No. 1 2 3 4 5 6 7 8 9 10 Total


Marks Obtained

x2 y2
1. A tangent to the ellipse   1 meets its director circle at P and Q. Then the product of the slopes
9 4
of CP and CQ where 'C' is the origin is
9 4 2 1
(A) (B) (C) (D) –
4 9 9 4

2. The equation of the circle whose diameter is the common chord of the circles ;
x² + y² + 3x + 2y + 1 = 0 & x² + y² + 3x + 4y + 2 = 0 is :
(A) x² + y² + 8x + 10y + 2 = 0 (B) x² + y²  5x + 4y + 7 = 0
(C) 2x² + 2y² + 6x + 2y + 1 = 0 (D) None of these

3. The area (in square unit) of the quadrilateral formed by the two pairs of lines
2x2 – m2y2 – n(x + my) = 0 and 2x2 – m2y2 + n(x – my) = 0 is

(A) n2 (B) n2 (C)


n
(D) n2
2| m| | m| 2| m| 4| m|

1
4. Line L1 is parallel to the line L2. Slope of L1 is 9. Also line L3 is parallel to line L4. Slope of L4 is . All
25
x2 y2
these lines touch the ellipse   1 . Then
25 9
(A) y = 9x + c where c2 = 2034 can be equation of L1
(B) x + 25y = p where p2 = 650 can be the equation of L3
(C) Area of the parallelogram formed by these lines is 5 square units.
(D) Area of the parallelogram formed by these lines is 60 square units.

5. Tangents are drawn from the point P(1, –1) to the circle x2 + y2 – 4x – 6y – 3 = 0 with centre C. A and B
are the points of contact. Which of the following are correct ?
(A) Area of quadrilateral PACB, is 4
(B) Area of triangle formed by pair of tangents and corresponding chord of contact, is 4/17
(C) Equation of the circle circumscribing the triangle formed by pair of tangent and corresponding
chord of contact, is x2 + y2 – 3x – 2y – 1 = 0
8
(D) Length of the chord of contact AB is
17

6. Equation x2 ex = k has

(A) Three real solutions if k  (0, 4e 2) (B) Two real solutions if k = 4e 2
(C) Only one real solution if k  (4e 2, ) (D) Only one real solution if k  (–, 0)

Reg & Corp. Office : CG Tower, A-46 & 52, IPIA, Near City Mall, Jhalawar Road, Kota (Raj.)-324005
Website: www.resonance.ac.in | E-mail : contact@resonance.ac.in
PAGE NO.-58
Toll Free : 1800 258 5555 | CIN: U80302RJ2007PLC024029
DPPs BOOKLET-1 VIJETA (01-02JPA) | MATHEMATICS

7. Given two straight lines x – y – 7 = 0 and x – y + 3 = 0. Equation of a line which divides the distance
between them in the ratio 3 : 2 (internally) can be :
(A) x – y – 1 = 0 (B) x – y – 3 = 0 (C) y = x (D) x – y + 1 = 0

8. The equations of the sides AB, BC, CA of a triangle ABC are 2x – y – 3 = 0, 6x + y – 21 = 0 and
2x + y – 5 = 0 respectively. The external bisector of angle A passes through the point
(A) (3, 1) (B) (4, 2) (C) (2, – 7) (D) (2, 1)

The function f(x) = ax  bx  cx  d has its non zero local minimum and maximum values at x = –2
3 2
9.
and x = 2 respectively. If a is a root of the equation x 2 – x – 6 = 0, then
(A) a = 3 (B) b = 0 (C) c = 24 (D) d = 32 a

x x
10. The value of a for which the function f(x) = (4a – 3) (x + log 5) + 2(a – 7) cot sin2 does not posses
2 2
critical points is ______.

DPP No. # A40 (JEE–MAIN)


Total Marks : 61 Max. Time : 60 min.
Single choice Objective ('-1' negative marking) Q. 1 to Q.19 (3 marks 3 min.) [57, 57]
Multiple choice objective ('-2' negative & Partial marking) Q.20 (4 marks 3 min.) [04, 03]

Ques. No. 1 2 3 4 5 6 7 8 9 10 11 12 13 14 15 16 17 18 19 20 Total


Mark
Obtained


1. Tangent and normal to the curve y = 2 sinx + sin2x are drawn at p  x  . The area of the quadrilateral
 3 
formed by the tangent, the normal and coordinate axes is.
 3  
(A) (B) (C) (D) None of these
2 2 2

2. The sides of a triangle ABC are as shown in the given figure. Let D be any internal point of this triangle
and let e, f, and g denote the distance between the point D and the sides of the triangle. The sum
(5e + 12f + 13g) is equal to

(A) 120 (B) 90 (C) 60 (D) 30

1
 a2 a3  
3. If sin  a   .....  + cos–1 (1 + b + b2 + ....) =
 , then the value of 'a' and 'b' may be
 3 9  2
1
(A) a = –3 & b = 1 (B) a = 1 & b = –1/3 (C) a = 6 & b  (D) None of these
2

4. For hyperbola x2 sec2 – y2 cosec2 = 1, which of the following remains constant with change in ''
(A) abscissae of vertices (B) abscissae of foci (C) eccentricity (D) directrix

5. If eccentricity of conjugate hyperbola of the hyperbola

Reg & Corp. Office : CG Tower, A-46 & 52, IPIA, Near City Mall, Jhalawar Road, Kota (Raj.)-324005
Website: www.resonance.ac.in | E-mail : contact@resonance.ac.in
PAGE NO.-59
Toll Free : 1800 258 5555 | CIN: U80302RJ2007PLC024029
DPPs BOOKLET-1 VIJETA (01-02JPA) | MATHEMATICS

 x – 1   y – 2  x – 5   y – 5
2 2 2 2
– = 3 is e, then value of 8e is

(A) 12 (B) 14 (C) 17 (D) 10

6. Consider points A(3, 4) and B(7, 13). If P be a point on the line y = x such that PA + PB is minimum,
then coordinates of P are
 12 12   13 13   31 31 
(A)  , (B)  , (C)  , (D) (0, 0)
 7 7   7 7   7 7 

7. If the line y = x cuts the curve x3 + 3y3 – 30xy + 72x – 55 = 0 in points A, B and C, then the value of
4 2
OA.OB.OC (where O is the origin), is
55
1
(A) 55 (B) (C) 2 (D) 4
4 2
8. Let 0 <  < /2 be fixed angle. If P = (cos, sin ) and Q = (cos( – ), sin ( – )), then Q is obtained
from P by
(A) clockwise rotation around origin through an angle 

(B) anticlockwise rotation around origin through an angle 


(C) reflection in the line through origin with slope tan 
(D) reflection in the line through origin with slope tan (/2)

9. Circles of radius 2 are centered at the vertices of a triangle with sides lengths 8, 11 and 12. Find the
length of a belt that fits tightly around those three circles as shown in the figure.


11 8
 
12
(A) 31 + 4 (B) 31 + 6 (C) 31 +  (D) 31 + 2

10. A circle is inscribed inside a regular pentagon and another circle is circumscribed about this pentagon.
Similarly a circle is inscribed in a regular heptagon and another circumscribed about the heptagon. The
area of the regions between the two circles in two cases are A1 and A2 respectively. If each polygon has
a side length of 2 units then which one of the following is true?
5 25 49
(A) A1 = A 2 (B) A1 = A2 (C) A1 = A2 (D) A1 = A2
7 49 25

11. If two chords of the circle x2 + y2  ax  by = 0, drawn from the point (a, b) is divided by the
x  axis in the ratio 2 : 1 then :
(A) a2 > 3 b2 (B) a2 < 3 b2 (C) a2 > 4 b2 (D) a2 < 4 b2

12. If mean of xi, xi+1 is xi+2  i  {1,2,3, ......} then lim xn equals to
n 

x1  x 2 2x1  x 2 x1  2x 2
(A) (B) (C) (D) None of these
2 3 3
13. Negation of the statement p  (q  r) is
(A) ~p  ~(q  r) (B) ~p  (q  r) (C) (q  r)  p (D) p  (~q  ~r)

14. The two curves x = y2 and xy = a3 cut orthogonally, then a2 is equal to


(A) 1/3 (B) 3 (C) 2 (D) 1/2
2 7
d (cos x)
15. If y = sin x, then is equal to
dy 2
Reg & Corp. Office : CG Tower, A-46 & 52, IPIA, Near City Mall, Jhalawar Road, Kota (Raj.)-324005
Website: www.resonance.ac.in | E-mail : contact@resonance.ac.in
PAGE NO.-60
Toll Free : 1800 258 5555 | CIN: U80302RJ2007PLC024029
DPPs BOOKLET-1 VIJETA (01-02JPA) | MATHEMATICS
(A) 35 cos3x – 42 cos5x (B) 35 cos3x
+ 42 cos5x
(C) 42 cos3x – 35 cos5 x (D) – 35 cos x – 42 cos5x
3

16. The gradient of the common tangent to the two curves y = x 2  5x + 6 and y = x2 + x + 1 is :
(A)  1/3 (B)  2/3 (C)  1 (D)  3
17. The point(s) of minimum of the function, f(x) = 4x  x x  2, x  [0, 3] is :
3

(A) x = 0 (B) x = 1/3 (C) x = 1/2 (D) x = 2

18. The equation to the chord joining two points (x 1, y1) and (x2, y2) on the rectangular hyperbola
xy = c2 is :

x y x y
(A) + =1 (B) + =1
x1  x 2 y1  y 2 x1  x 2 y1  y 2
x y x y
(C) + =1 (D) + =1
y1  y 2 x1  x 2 y1  y 2 x1  x 2

19. A point on a parabola y2 = 4ax, the foot of the perpendicular from it upon the directrix, and the focus are
the vertices of an equilateral triangle. The focal distance of the point is equal to -
(A) a/2 (B) a (C) 2a (D) 4a

20. Identify the correct statement


(A) If a, b, c are odd integers a + b + c cannot be zero
(B) If a, b, c are odd integers a2 + b2 – c2  0
(C) If a2 + b2 = c2, then at least one of a, b, c is even, given that a, b, c are integers
(D) If a2 + b2 = c2 where a, b, c are integers then c > a + b

DPP No. # A41 (JEE–ADVANCED)


Revision DPP on "Circle"
Total Marks : 35 Max. Time : 30 min.
Comprehension ('–1' negative marking) Q. 1 to Q.2 (3 marks 3 min.) [06, 06]
Single choice Objective ('-1' negative marking) Q. 3 to Q.5 (3 marks 3 min.) [09, 09]
Multiple choice objective ('-2' negative & Partial marking) Q.6 to Q.10 (4 marks 3 min.) [20, 15]

Question No. 1 2 3 4 5 6 7 8 9 10 Total


Marks Obtained

Comprehension # 1 (Q. No 1 & 2).


Let AB be any chord of the circle x² + y² – 2x – 6y – 6 = 0 which subtends a right angle at the point
(2, 4).

1. Locus of mid point of AB is


(A) x² + y² – 3x – 7y –16 = 0 (B) x² + y² – 3x – 7y + 7 = 0
(C) x² + y² + 3x + 7y – 16 = 0 (D) x² + y² + 3x + 7y – 7 = 0

2. Locus of the point of intersection of perpendicular tangents on the curve obtained in above question is
(A) 2x2 + 2y2 – 4x – 7y – 1 = 0 (B) 2x2 + 2y2 – 14x – 7y – 1 = 0
(C) 2x + 2y – 6x – 14y – 1 = 0
2 2
(D) 2x2 – 12y2 – 6x + 14y – 1 = 0

3. If ax + by – 5 = 0 is the equation of the shortest chord of the circle (x – 3)2 + (y – 4)2 = 4 passing
through the point (2, 3), then | a + b | is
(A) 2 (B) 4 (C) 1 (D) 8

4. A circle touches the sides AB and AD of a rectangle ABCD at P and Q respectively and passes through
the vertex C. If the distance of C from chord PQ is 5 units, then area of the rectangle is
(A) 45 (B) 25 (C) 50 (D) 75
Reg & Corp. Office : CG Tower, A-46 & 52, IPIA, Near City Mall, Jhalawar Road, Kota (Raj.)-324005
Website: www.resonance.ac.in | E-mail : contact@resonance.ac.in
PAGE NO.-61
Toll Free : 1800 258 5555 | CIN: U80302RJ2007PLC024029
DPPs BOOKLET-1 VIJETA (01-02JPA) | MATHEMATICS

5. The circles having radii r1 and r2 intersect orthogonaly. Length of their common chord is
2 r1 r2 2 r12 r2 r1 r2 2 r22 r2
(A) (B) (C) (D)
r12  r22 r12  r22 r12  r22 r12  r22

6. A circle with centre in the first quadrant is tangent to y = x + 10, y = x – 6 and the y-axis. If the equation
of the circle is (x – h)2 + (y – k)2 = r2 then

(A) h = 2 2 (B) k = 4 2+2 (C) r = 8 2 (D) h + k = 2 + 8 2


7. Two circles touch the x-axis and the line y = mx (m>0). They meet at (9, 6) and at another point and the

product of their radii is 68 and m = 221 where  &  are relatively prime numbers, then

(A)  = 12 (B) is perfect square


(C)  –  is prime number (D)  is divisible by 7

8. If Line L : (3x – 4y – 25 = 0) touches the circle S : (x2 + y2 – 25 = 0) at P and L is common tangent of


circles S = 0 and S1 = 0 at P and S1 = 0 passes through (5, –6), then

(A) centre of S1 = 0,  ,  
27 36
 7 7 

275
(B) length of tangent form origin to S1 = 0,
7

(C) centre of S1 = 0,  ,
27 36 
7 7  

375
(D) length of tangent from origin to S1 = 0 =
7

9. Consider the circle x2 + y2 – 8x – 18y + 93 = 0 with centre 'C' and point P(2, 5) outside it.
From the point P, a pair of tangents PQ and PR are drawn to the circle with S as the midpoint of
QR. The line joining P to C intersects the given circle at A and B. Which of the following hold(s)
good?
(A) CP is the arithmetic mean of AP and BP.
(B) PR is the geometric mean of PS and PC.
(C) PS is the harmonic mean of PA and PB.

(D) The angle between the two tangents from P is tan –1   .


3
4

10. Let the straight line L : x – 2y = 8, be rotated, through an angle '' (where tan  = 1/3), about the point
P(0, –4) in anticlockwise sence. After rotation the line becomes tangent to the circle which lies in
4th quadrant and also touches co-ordinate axes. Which of the following is/are correct ?
(A) Radii of all the possible circles are the roots of the equation r 2 – 8r + 8 = 0
(B) After rotation equation of new line is x– y – 4 = 0
Reg & Corp. Office : CG Tower, A-46 & 52, IPIA, Near City Mall, Jhalawar Road, Kota (Raj.)-324005
Website: www.resonance.ac.in | E-mail : contact@resonance.ac.in
PAGE NO.-62
Toll Free : 1800 258 5555 | CIN: U80302RJ2007PLC024029
DPPs BOOKLET-1 VIJETA (01-02JPA) | MATHEMATICS

(C) Difference of the radii of the possible circles is 4 2 .


(D) Area of one of the possible circle is 8(3  2 2 ) sq. units

DPP No. # A42 (JEE–ADVANCED)


Total Marks : 35 Max. Time : 30 min.
Single choice Objective ('-1' negative marking) Q. 1 to Q.5 (3 marks 3 min.) [15, 15]
Multiple choice objective ('-2' negative & Partial marking) Q.6 to Q.10 (4 marks 3 min.) [20, 15]
Question No. 1 2 3 4 5 6 7 8 9 10 Total
Marks Obtained

1. In the given figure PQ is tangent the circle at P and length of PQ is equal to the arc AP, then
co-ordinate of point Q is-

Q
/4
O

A(1, 0)

FG   4 ,   4 IJ FG 4   , 4   IJ FG 4   , 4   IJ FG   4 , 4   IJ
(A)
H4 2 4 2K (B)
H4 2 4 2K (C)
H4 2 4 2K (D)
H4 2 4 2K
2. A straight line with slope 2 and y-intercept 5 touches the circle x2 + y2 + 16x + 12y + c = 0 at a point Q.
Then the coordinates of Q are
(A) (–6, 11) (B) (–9, –13) (C) (–10, –15) (D) (–6, –7)

3. The number of distinct pairs (x, y) of the real number satisfying x = x 3 + y4 and y = 2xy is :
(A) 5 (B) 12 (C) 3 (D) 7

4. Let f(x) : R+  R+ is an invertible function such that f (x) > 0 and f (x) > 0  x  [1, 5]. If f(1) = 1 and
f(5) = 5 and area under the curve y = f(x) on x-axis from x = 1 to x = 5 is 8 sq. units, then area bounded
by y = f–1(x) on x-axis from x = 1 to x = 5 is
(A) 8 (B) 12 (C) 16 (D) 20

5. As shown in the figure, circles with centres A, B and C are externally tangent to each other and
internally tangent to the circle with centre D. Circles B and C are congruent. Circle A has radius 1 and
passes through D. The radius of the circle B, is

3 7 8 1 3
(A) (B) (C) (D)
2 8 9 3

  
2, 

6. f(x) = cos (lnx), g(x) = ln (cosx), x   e  then identify the INCORRECT statement-
 2
 
Reg & Corp. Office : CG Tower, A-46 & 52, IPIA, Near City Mall, Jhalawar Road, Kota (Raj.)-324005
Website: www.resonance.ac.in | E-mail : contact@resonance.ac.in
PAGE NO.-63
Toll Free : 1800 258 5555 | CIN: U80302RJ2007PLC024029
DPPs BOOKLET-1 VIJETA (01-02JPA) | MATHEMATICS
(A) f(x) > g(x) (B) g(x) > f(x)
(C) g(x) = f(x) has 2 solutions (D) f(x) = g(x) has 4 solutions

7. f(x) is a cubic polynomial with it's leading coefficient 'a'. x = 1 is a point of extremum of f(x) and x = 2 is a
point of extremum of f'(x). Then
(A) The other point of extremum is at x = 3 (B) The other point of extremum is at x = 0
(C) If at x = 1, f(x) has a local maximum, a > 0 (D) If at x = 1, f(x) has a local minimum, a < 0

8. Consider the curve 4y = x2 and P(0, p), Q is a point on this parabola such that PQis minimum, then
(A) If p  2 then Q is (0, 0) (B) Q is (0, 0) p R

(2 p  2, p  2)  p2
(C) If p  2 then Q is (D) If p  2 then Q is  p  2,
 4 
9. At a point P on the parabola y2 = 4ax, tangent and normal are drawn. Tangent intersects the x-axis at Q
and normal intersects the curve at R such that chord PR subtends an angle of 90° at its vertex, then
(A) PQ = 2a 6 (B) PR = 6a 3
(C) Area of PQR = 18 2 a2 (D) PQ = 3a 2
10. A circle having its centre at (2, 3) is cut orthogonally by the parabola y2 = 4x. The possible intersection
point(s) of these curves, can be
(A) (3, 2 3 ) (B) (2, 2 2 ) (C) (1, 2) (D) (4, 4)

DPP No. # A43 (JEE–ADVANCED)


Total Marks : 40 Max. Time : 35 min.
Single choice Objective ('-1' negative marking) Q. 1 to Q.3 (3 marks 3 min.) [09, 09]
Multiple choice objective ('-2' negative & Partial marking) Q.4 to Q.8 (4 marks 3 min.) [20, 15]
Subjective Questions ('–1' negative marking) Q.9 (3 marks 3 min.) [03, 03]
Match the Following (each row '(2, -1)' negative marking) Q.10 (8 marks, 8 min.) [08, 08]
Question No. 1 2 3 4 5 6 7 8 9 10 Total
Marks Obtained

1. If g(x) = 2f (2x3 – 3x2) + f (6x2 – 4x3 – 3)  x  R and f(x) > 0,  x  R, then g(x) is increasing in the
interval
(A)  ,   and (0, 1) (B)   ,0  and (1,  ) (C) (0, )
1 1
(D) None of these
 2  2 

2. Two circles are externally tangent. Lines PAB and PA'B' are common tangents with A and A' on the
smaller circle and B and B' on the larger circle. If PA = AB = 4, then area of the smaller circle, is

(A) 1.44 (B) 2 (C) 2.56 (D) 

3. The lines 3x + 4y = 9 & 4x  3y + 12 = 0 intersect at P . The first line intersects xaxis at A and the
second line intersects yaxis at B . Then the circumradius of the triangle PAB is :
(A) 3/2 (B) 5/2 (C) 10 (D) none

4. Points in the interval [0, 2], where f(x) = ex.sinx has maximum or minimum slope is
  3
(A) (B) (C)    (D)
4 2 2

Reg & Corp. Office : CG Tower, A-46 & 52, IPIA, Near City Mall, Jhalawar Road, Kota (Raj.)-324005
Website: www.resonance.ac.in | E-mail : contact@resonance.ac.in
PAGE NO.-64
Toll Free : 1800 258 5555 | CIN: U80302RJ2007PLC024029
DPPs BOOKLET-1 VIJETA (01-02JPA) | MATHEMATICS
5. 2
Let a and b be two integers such that 10 a + b = 20 and g(x) = x + ax + b. If g(10) g(11) = g(n) and
x & y are the number of ways in which 'n' can be expressed as product of two factors and product of
two relatively prime factors , then
(A) x = 4 (B) x – y = 4 (C) y = 4 (D) x = 2y
x y x y
6. Line  = 1 cuts the coordinate axes at A(a, 0) & B (0, b) & the line  =  1 at
a b a b
A (a, 0) & B(0, b) . If the points A, B, A, B are concyclic then the orthocentre of the triangle ABA
is:
 aa   bb ' 
(A) (0, 0) (B) (0, b') (C)  0 , (D)  0 ,
 b   a 
7. Let A, B and C be three distinct points on y2 = 8x such that normals at these points are concurrent at P.
4
The slope of AB is 2 and abscissa of centroid of ABC is . Which of the following is (are) correct?
3
(A) area of ABC is 8 sq. units (B) coordinates of P are (6, 0)
(C) angle between normals are 45°, 45°, 90° (D) angle between normals are 30°, 30°, 60°

8. Let the straight line L : x – 2y = 8, be rotated, through an angle '' (where tan  = 1/3), about the point
P(0, –4) in anticlockwise sence. After rotation the line becomes tangent to the circle which lies in 4th
quadrant and also touches co-ordinate axes. Which of the following is/are correct ?
(A) Radii of all the possible circles are the roots of the equation r 2 – 8r + 8 = 0
(B) After rotation equation of new line is x– y – 4 = 0
(C) Difference of the radii of the possible circles is 4 2 .
(D) Area of one of the possible circle is 8(3  2 2 ) sq. units

9. Given right triangle ABC, with AB = 4, BC = 3 and CA = 5. Circle,  ,passes through A and is tangent to
BC at C. What is the radius of  ?

10. If P(x) = x3 + px2 + qx + 6, then match the entries in column - I with column - II
Column - I Column - II
(A) If P(x) is divisible by x + ax + b and x + bx + a,
2 2
(p) have point of local maximum
(a, b,  R), a  b, then P(x) less than point of local
minimum
(B) If 3q > p2, then P(x) (q) is monotonic  x  R
(C) If p and q are two consecutive natural numbers (r) has point of local maximum
such that p > q, then P(x) greater than point of local
minimum
(D) If Q(x) = P(x) – 2x3 – 2qx and p2 > 3q, then Q(x) (s) possesses local
maxima and local minima

DPP No. # A44 (JEE–MAIN)


Total Marks : 60 Max. Time : 60 min.
Single choice Objective ('-1' negative marking) Q. 1 to Q.20 (3 marks 3 min.) [60, 60]

Ques. No. 1 2 3 4 5 6 7 8 9 10 11 12 13 14 15 16 17 18 19 20 Total


Mark
Obtained

x2 y2
1. The minimum area of triangle formed by the tangent to the ellipse 2
+ = 1 and coordinate axes is
a b2
a2  b2 (a  b)2 a2  ab  b2
(A) ab sq. units (B) sq. unit (C) sq. units (D) sq. units
2 2 3

2. A pair of tangents are drawn to a unit circle with centre at the origin and these tangents intersect at the
point P enclosing an angle of 600. The area enclosed by the tangents and the arc of the circle is,
Reg & Corp. Office : CG Tower, A-46 & 52, IPIA, Near City Mall, Jhalawar Road, Kota (Raj.)-324005
Website: www.resonance.ac.in | E-mail : contact@resonance.ac.in
PAGE NO.-65
Toll Free : 1800 258 5555 | CIN: U80302RJ2007PLC024029
DPPs BOOKLET-1 VIJETA (01-02JPA) | MATHEMATICS
2    3  
(A)  (B) 3 (C)  (D) 3 1  
3 6 3 3 6  6

3. If the radius of the circumcircle of the triangle TPQ, where PQ is chord of contact corresponding to point
T with respect to circle x2 + y2 – 2x + 4y – 11 = 0, is 6 units, then distance of T from the director circle of
the given circle is :
(A) 6   (B) 12  (C) 6 2 (D) 12 – 4 2
4. A point 'P' is an arbitrary interior point of an equilateral triangle of side 4. If x, y, z are the
distances of 'P' from sides of the triangle then the value of (x + y + z) 2 is equal to
(A) 3 (B) 12 (C) 18 (D) 48

5. If  ,  ,  are the real roots of the equation , x3  3 p x2 + 3 q x  1 = 0 , the centroid of the triangle
 1  1
whose vertices are   ,
1
,  , ,  ,  is :
      
(A) (p , p) (B) (p , 0) (C) (p , q) (D) (q , 0)
6. The algebraic sum of perpendicular distances from A (x 1, y1), B (x2, y2) and C (x3, y3) to a variable line is
zero, then all the such lines will always pass through _______
(A) the orthocentre of ABC (B) the centroid of ABC
(C) the circumcentre of ABC (D) the incentre of ABC
7. The angle bisectors of angle A for the triangle ABC whose coordinates of the vertices
are A(–8, 5) ; B(–15, –19) and C(1, –7) has the equations
(A) 11 x + 2y + 3 = 0 (B) 2x – 11y + 71 = 0 (C) 11x + 2y + 78 = 0 (D) 2x – 11y + 7 = 0

8. If variance of ten observations 10, 20, 30, 40, 50, ....... 100 is A and variance of ten another
B
observations 22, 42, 62, 82, 102, ......., 202 is B, then is
A
(A) 0 (B) 1 (C) 2 (D) 4

9. The curve y = f(x) which satisfies the condition f  (x) > 0 and f  (x) < 0 for all real x, is:
y y y
y

(A) (B) (C) (D)


x
x x 0 x
0 0 0

10. Let f be differentiable (x  R). if f(1) = – 2 and f(x)  2 for x  [1, 6], then
(A) f(6) < 6 (B) f(6)  8 (C) f(6) = 5 (D) f(6)  5

 f(a  h) 
11. If lim  = c where f(x) is a continuous function such that f(x) > 0 for all x  R and [ . ] denotes
h0  f(a) 
greatest integer function then which of the following statements is always true ?
(A) If x = a is a point of local minima then c  N
(B) If x = a is a point of local maxima then c  N
(C) If x = a is a point of local minima then c  –
(D) If x = a is a point of local maxima then c  –
4
12. Rolle's Theorem holds for the function f(x) = x 3 + bx2 + cx, 1  x  2 at x = , then the value of
3
b + c is
(A) 2 (B) 3 (C) 4 (D) 5

Reg & Corp. Office : CG Tower, A-46 & 52, IPIA, Near City Mall, Jhalawar Road, Kota (Raj.)-324005
Website: www.resonance.ac.in | E-mail : contact@resonance.ac.in
PAGE NO.-66
Toll Free : 1800 258 5555 | CIN: U80302RJ2007PLC024029
DPPs BOOKLET-1 VIJETA (01-02JPA) | MATHEMATICS
2 2
13. The point of intersection of tangents drawn to the hyperbola x 2 – y 2 = 1 at the points where it is
a b
intersected by the line x + my + n = 0, is
 a2 b2m   a2 b2n   a2 b2n   a 2 b 2n 
(A)  ,  (B)  ,  (C)  ,  (D)  , 
 n n   m m   m m   m m 
14. A normal is drawn at the point P (3, 6) on the parabola y2 = 12x, intersecting the curve again at Q.
Equation of circle on this normal chord PQ as diameter is
(A) x2 + y2 – 30x + 12y – 27 = 0 (B) x2 + y2 – 30x – 12y + 117 = 0
(C) x2 + y2 – 15x + 6y – 36 = 0 (D) x2 + y2 + 24x + 12y – 189 = 0

15. The number of integral values of k for which the chord of the circle x 2 + y2 = 125 passing through the
point R(8, k) gets bisected at the point P(8, k) and has integral slope is
(A) 8 (B) 6 (C) 4 (D) 2

16. Let P and Q be the subsets of the X-Y plane defined as :


P= {(x, y) : x > 0, y > 0 and x2 + y2 = 1},
and Q= {(x, y) : x > 0, y > 0 and x8 + y8 < 1},
Then, P  Q is-
(A) the empty set  (B) P (C) Q (D) none of the foregoin sets.

17. Let U = {x  N ; x  10}, A = {x  N ; 1  x3 < 150} and B = {x  N ; x is prime number greater than 2}
and C = {x  N ; 5x} then number of elements in (Ac  Bc  Cc) is
(A) 3 (B) 4 (C) 8 (D) 5

18. If the lines x2 + 2xy – 35y2 – 4x + 44y – 12 = 0 and 5x + y – 8 = 0 are concurrent, then the value of
is.
(A) 0 (B) 1 (C) – 1 (D) 2

19. Consider the following statements

Statement-1: The area of the triangle formed by the points A(20, 22); B(21, 24) and C(22, 23) is the
same as the area of the triangle formed by the point P(0, 0); Q(1, 2) and R(2, 1).
because
Statement-2: The area of the triangle is invariant w.r.t. the translation of the coordinate axes.
(A) Statement-1 is true, statement-2 is true and statement-2 is correct explanation for statement-1.
(B) Statement-1 is true, statement-2 is true and statement-2 is NOT the correct explanation for
statement-1.
(C) Statement-1 is true, statement-2 is false.
(D) Statement-1 is false, statement-2 is true.
20. The number of integer values of m, for which the x-coordinate of the point of intersection of the lines,
3x + 4y = 9 and y = mx + 1 is also an integer is
(A) 2 (B) 0 (C) 4 (D) 1

DPP No. # A45 (JEE–ADVANCED)


Revision DPP on "Conic Section"
Total Marks : 37 Max. Time : 30 min.
Single choice Objective ('-1' negative marking) Q. 1 (3 marks 3 min.) [03, 03]
Multiple choice objective ('-2' negative & Partial marking) Q.2 to Q.8 (4 marks 3 min.) [28, 21]
Subjective Questions ('–1' negative marking) Q.9 to Q.10 (3 marks 3 min.) [06, 06]
Question No. 1 2 3 4 5 6 7 8 9 10 Total
Marks Obtained
1. An endless inextensible string of length 15m passes around two pins, A & B which are 5m apart. This
string is always kept tight and a small ring, R, of negligible dimensions, inserted in this string is made to
Reg & Corp. Office : CG Tower, A-46 & 52, IPIA, Near City Mall, Jhalawar Road, Kota (Raj.)-324005
Website: www.resonance.ac.in | E-mail : contact@resonance.ac.in
PAGE NO.-67
Toll Free : 1800 258 5555 | CIN: U80302RJ2007PLC024029
DPPs BOOKLET-1 VIJETA (01-02JPA) | MATHEMATICS
move in a path keeping all segments RA, AB, RB tight (as mentioned earlier). The ring traces a path,
given by conic C, then
1
(A) Conic C is an ellipse with eccentricity (B) Conic C is an hyperbola with eccentricity 2
2
2 3
(C) Conic C is an ellipse with eccentricity (D) Conic C is a hyperbola with eccentricity
3 2
2. A circle having its centre at (2, 3) is cut orthogonally by the parabola y2 = 4x. The possible intersection
point(s) of these curves, can be
(A) (3, 2 3 ) (B) (2, 2 2 ) (C) (1, 2) (D) (4, 4)

3. If a conic passing through origin has (3, 3),(– 4, 4) as its focii, then
(A) auxillary circle is (2x + 1)2 + (2y – 7)2 = 2 (B) auxillary circle is (2x + 1)2 + (2y – 7)2 = 98
(C) auxillary circle is (2x + 1)2 + (2y – 1)2 = 49 (D) auxillary circle is (2x + 1)2 + (2y – 1)2 = 1

4. Let C be the centre of an ellipse and P is any point on it. Tangent at P meets the major and minor axes
at T and t respectively. G is point of intersection of normal at P, with major axis. If perpendicular from P
on major and minor axes are PM and PN such that CM.CT = 16, CN.Ct = 9, then
9
(A) the length of latus rectum is . (B) the length of latus rectum is 9.
2
(C) MG : MC = 9 : 16. (D) MG : MC = 3 : 4.

5. Equation (5x – 1)2 + (5y – 2)2 = (2 – 2 + 1) (3x + 4y – 1)2 represents 
(A) an ellipse if (0, 1)  (1, 2) (B) a parabola if {0, 2}
(C) a hyperbola if  < 0 (D) a hyperbola if  = 4

6. A parabola has its vertex and focus in the first quadrant and axis along the line y = x. If the distances of
the vertex and focus from the origin are respectively 2 and 2 2 , then equation of the parabola is
(A) x2 + y2 – 8x + 8y + 2xy = 16 (B) x2 + y2 – 8x – 8y + 16 = 2xy
(C) (x – y) = 8(x + y – 2)
2 (D) (x + y)2 = 8(x – y + 2)

7. The equation of a parabola is (x + y – 2)2 = 2 (x – y) then


(A) Vertex of the parabola is (1,1)
(B) Latus rectum of parabola is 2.
(C) The equation of tangent at vertex is x + y – 2 = 0
(D) Foot of perpendicular from focus on any tangent of the parabola lies on x – y = 0.

8. The locus of the point of intersection of two mutually perpendicular normals of the parabola
x2 = 8 y is a parabola. Which of the following hold(s) good in respect of the locus?
(B) Coordinates of focus is  0,
11 
(A) Length of the latus rectum is 2. 
 2 
(C) Equation of a director circle is 2y – 11 = 0 (D) Equation of axis of symmetry is y = 0.

9. Maximum area of the triangle formed by a moving point p on the ellipse and focii, S and S', is 1. Point

Q is an extremity of latus rectum. 2 is the area formed by points Q, S and S'. Given that 1  3 , latus
2
4 x2 y2 a2 ?
rectum of the ellipse is and equation of the ellipse is   1 (a > b > 0), find the value of
3 a2 b2 b2
Reg & Corp. Office : CG Tower, A-46 & 52, IPIA, Near City Mall, Jhalawar Road, Kota (Raj.)-324005
Website: www.resonance.ac.in | E-mail : contact@resonance.ac.in
PAGE NO.-68
Toll Free : 1800 258 5555 | CIN: U80302RJ2007PLC024029
DPPs BOOKLET-1 VIJETA (01-02JPA) | MATHEMATICS

10. An ellipse with focii (1, 4) and (,) touches x-axis at (5, 0). Then value of (– ) is

Reg & Corp. Office : CG Tower, A-46 & 52, IPIA, Near City Mall, Jhalawar Road, Kota (Raj.)-324005
Website: www.resonance.ac.in | E-mail : contact@resonance.ac.in
PAGE NO.-69
Toll Free : 1800 258 5555 | CIN: U80302RJ2007PLC024029
DPPs BOOKLET-1 VIJETA (01-02JPA) | MATHEMATICS

ANSWER KEY
DPP No. # A1 DPP No. # A17
10. x = –9, 0, –24, 15 10 (A) Q; (B)  P; (C)  S; (D)  R

DPP No. # A2 DPP No. # A19


9. x = 30° 10. 1 10. 4

DPP No. # A3 DPP No. # A22


10. 30 9. {(45, 180), (180, 45) , (80, 80)}
10. /4
DPP No. # A4
20. 4904 DPP No. # A23
8. 1 9. 2
DPP No. # A5
x   tan , cot1
1
10.
a  2 
10.
4
DPP No. # A24
20. 4
DPP No. # A6
10. 4
DPP No. # A25
DPP No. # A7 8.
17
9. 37 10. 0002
9. 2002000 24
10. (i) 6720 (ii) 2880 (iii) 12p4
DPP No. # A26
DPP No. # A9 2. 469 3. 27 4. 9
5. 14 6. 3
7. 2007 + /2
7. 16800
8. (A)  (s) ; (B)  (q) ; (C)  (r) ; (D)  (p)
8. (i)-1008, (ii) -320, (iii)-68, (iv)-168, (v)-97
10. –50C25
DPP No. # A10
8. 3 9. 46
DPP No. # A29
10. [(A) S; (B) P; (C) Q; (D) R]
7. 11 8. 6 9. n = 18
10. x = 0, y = 0, z = 0
DPP No. # A12
10. 1002C
50 DPP No. # A30
9. 54 10. 10
DPP No. # A13
 5 DPP No. # A31
10 and
12 12 2 5
15. (i) + + 7 sec2x
1
x2
DPP No. # A15 3x 3
(ii) ex x (2 n x + 1 + x n x)
9. 34
10. A-PR ; B-PR ; C-QS ; D-QS (iii) sec x
x2
(iv)
DPP No. # A16 (x sin x  cos x )2
10. (A)(q,r),(B)(r,s),(C)(p,q),(D)(p,s, t) 1 x
(v) sec2
2 2
Reg & Corp. Office : CG Tower, A-46 & 52, IPIA, Near City Mall, Jhalawar Road, Kota (Raj.)-324005
Website: www.resonance.ac.in | E-mail : contact@resonance.ac.in
PAGE NO.-70
Toll Free : 1800 258 5555 | CIN: U80302RJ2007PLC024029
DPPs BOOKLET-1 VIJETA (01-02JPA) | MATHEMATICS

16.
 cos x
(i) (n x)cos x 

 sin x n( nx) 
DPP No. # A34
 x nx  8. f(x) is discontinuous at x = 6 and non-

(ii) xx (1 + n x) – n2 . 2sinx . cos x 9  21


differentiable at x = ,6
2
(iii) (x n x)n n x.

1   2  (0,6)
18
 1  n ( n x) 1  
x   n x   6
0 1 9  2 21 5
17. (2, 3]
18. (i) tan t (ii) – tan 3t
–19
ax  hy  g
19. (i) –
hx  by  f
9. 1 10. 69
2x  y  e y  e x y
(ii)
x  xe  y  e x
DPP No. # A36

20. (i)
dy
=
5 d2 – d  1
7. 2
dx (1  25x 2 ) 3
dy 2 8. x2 + y2 – 7x + 5y = 0.
(ii) =–
dx 1  x2 9. (m2 – 1) x – my + b(m2 + 1) + am = 0
1 10. 4
(iii)
2 1 x 2

DPP No. # A37


DPP No. # A32
10. 2
b
13.
ab  2ay
DPP No. # A38
(n  1)xn2  (n  2)xn1  1
14. 9. (2  2)
(1  x)2

1 1 10. A-S ; B-P ; C-R ; D- P


16. (i) (ii)
2 x 1 4 x 2 2
DPP No. # A39
DPP No. # A33 10. (– , – 4/3) [2, )

15. cosec2 x – (1/x2)


DPP No. # A43
9. 25/8
10. (A)p, s; (B) q ; (C) p, s ; (D) r, s

DPP No. # A45


9. 9 10. 5

Reg & Corp. Office : CG Tower, A-46 & 52, IPIA, Near City Mall, Jhalawar Road, Kota (Raj.)-324005
Website: www.resonance.ac.in | E-mail : contact@resonance.ac.in
PAGE NO.-71
Toll Free : 1800 258 5555 | CIN: U80302RJ2007PLC024029

You might also like